Расчет балок на изгиб онлайн: СОПРОМАТ ГУРУ. Расчет балки онлайн. Построение эпюр

Содержание

Расчет балки

Подробный ход решения — расчет балки, построение эпюр

Заменим распределенную нагрузку равнодействующей

Q1 = 6·2 = 12кН

Составим уравнения равновесия для определения реакций опор

Σ MA = + P · 2 + M + Q1 · 3 — RE · 6= + 12 · 2 + 8 + 12 · 3 — RE · 6=0

Σ ME = — P · 4 + M — Q1 · 3 + RA · 6= — 12 · 4 + 8 — 12 · 3 + RA · 6=0

Из этих уравнений находим реакции опор

RA = 12.67кН.

RE = 11.33кН.

Записываем уравнения поперечных сил и изгибающих моментов на участках балки , используя метод сечений

На участке AB: (0 ≤ z1 ≤ 2 м )

Q(z1) = + RA = + 12.67 = 12.667 кН

M(z1) = + RA · z = + 12.67 · z

M(0) = 0 кНм

M(2) = 25.333 кНм

На участке BC: (2 ≤ z2 ≤ 4 м )

Q(z2) = + RA — P — q1·(z — 2) = + 12.

67 — 12 — 6·(z — 2)

Q(2) = 0.667 кН

Q(4) = -11.333 кН

M(z2) = + RA · z — P·(z — 2) — q1·(z — 2)2/2 = + 12.67 · z — 12·(z — 2) — 6·(z — 2)2/2

M(2) = 25.333 кНм

M(4) = 14.667 кНм

Поскольку поперечная сила на участке пересекает ноль при z = 2.11 м, в этой точке будет экстремум на эпюре M

M(2.11) = 25.4 кНм

На участке CD: (4 ≤ z3 ≤ 5 м )

Q(z3) = + RA — P — Q1 = + 12.67 — 12 — 12 = -11.333 кН

M(z3) = + RA · z — P·(z — 2) — Q

1·(z — 3) = + 12.67 · z — 12·(z — 2) — 12·(z — 3)

M(4) = 14.667 кНм

M(5) = 3.333 кНм

На участке DE: (5 ≤ z4 ≤ 6 м )

Q(z4) = + RA — P — Q1 = + 12.67 — 12 — 12 = -11.333 кН

M(z4) = + RA · z — P·(z — 2) + M — Q1·(z — 3) = + 12.67 · z — 12·(z — 2) + 8 — 12·(z — 3)

M(5) = 11. 333 кНм

M(6) = 0 кНм

Максимальный момент в балке составляет Mmax = 25.4 кНм. По этому значению подбираем сечение балки.

Условие прочности при изгибе σ = Mmax / W ≤ [σ]

Отсюда, минимально необходимый момент сопротивления вычисляем по формуле

Wmin=Mmax / [σ]

Расчет опорных реакций балки на двух опорах онлайн

Расчет выполняется по следующей методике:

1. Заменяем распределенную нагрузку ее равнодействующей, которая является сосредоточенной силой. Для равномерно распределенной нагрузки равнодействующая равна произведению интенсивности нагрузки q на длину участка L, на котором она действует: Fq = q*L.

2. Обозначаем опоры. Общепринято их обозначать буквами А и В. Простая балка имеет одну шарнирно-неподвижную и одну шарнирно-подвижную опоры.

3. Освобождаемся от опор и заменяем их действие на балку реакциями.
Реакции опор при такой нагрузке будут только вертикальными.

4. Составляем уравнения равновесия вида:
MA = 0; MB = 0,
Моментом силы относительно точки называется произведение этой силы на плечо — кратчайшее расстояние от этой точки приложения силы (в общем случае — до линии действия силы).

5. Выполним проверку решения. Для этого составим уравнение равновесия: Y = 0,
Если оно удовлетворено, то реакции найдены правильно, а если нет, но в решении допущена ошибка.

6. Строим эпюру поперечных сил Q

x. Для этого определяем значения поперечных сил в характерных точках. Напомним, что поперечная сила в сечении равна сумме проекций всех сил, расположенных только слева или только справа от рассматриваемого сечения, на ось, перпендикулярную оси элемента. Силу, расположенную слева от рассматриваемого сечения и направленную вверх, считают положительной (со знаком «плюс»), а направленную вниз — отрицательной (со знаком «минус»). Для правой части балки — наоборот.
В сечениях, соответствующих точкам приложения сосредоточенных сил, в том числе в точках приложения опорных реакций, необходимо определить два значения поперечной силы: чуть левее рассматриваемой точки и чуть правее ее.
Поперечные силы в этих сечениях обозначаются соответственно Qлев и Qправ.
Найденные значения поперечных сил в характерных точках откладываются в некотором масштабе от нулевой линии. Эти значения соединяются прямыми линиями по следующим правилам:
а) если к участку балки нет распределенной нагрузки, то под этим участком значения поперечных сил соединяются прямой линией, параллельной нулевой линии;
б) если на участке балки приложена распределенная нагрузка, то под этим участком значения поперечных сил соединяются прямой, наклонной к нулевой линии. Она может пересекать или не пересекать нулевую линию.
Соединив все значения поперечных сил по указанным правилам, получим график изменения поперечных сил по длине балки. Такой график называется эпюрой Q
x
.

7. Строим эпюру изгибающих моментов Мx. Для этого определяем изгибающие моменты в характерных сечениях. Напомним, что изгибающий момент в рассматриваемом сечении равен сумме моментов всех сил (распределенных, сосредоточенных, в том числе и опорных реакций, а также внешних сосредоточенных моментов), расположенных только слева или только справа от этого сечения. Если любое из перечисленных силовых воздействий стремится повернуть левую часть балки по часовой стрелке, то оно считается положительным (со знаком «плюс»), если против — отрицательным (со знаком «минус»), а для правой части наоборот.

В сечениях, соответствующих точкам приложения сосредоточенных моментов, необходимо определить два значения изгибающего момента: чуть левее рассматриваемой точки и чуть правее ее. Изгибающие моменты в этих точках обозначаются соответственно Млев и Мправ. В точках приложения сил определяется одно значение изгибающего момента.
Полученные значения откладываются в некотором масштабе от нулевой линии. Эти значения соединяются в соответствии со следующими правилами:
а) если на участке балки нет распределенной нагрузки, то под этим участком балки два соседних значения изгибающих моментов соединяются прямой линией;
б) если к участку балки приложена распределенная нагрузка, то под этим участком значения изгибающих моментов для двух соседних точек соединяются по параболе.

Пример решения балки:

Расчет балок часть 1 | Онлайн калькулятор

В данном разделе можно выполнить онлайн расчеты статически определимых балок в условиях прямого поперечного изгиба под действием сосредоточенной нагрузки. Расчеты определяют прогиб, угол поворота и изгибающий момент в произвольно заданной точке балки при различных граничных условиях. Определив наибольший изгибающий момент и соответствующее опасное сечение балки легко подобрать его размеры исходя из допускаемых напряжений в сечении.

Исходные данные:

L – длина балки, в миллиметрах;

a – координата точки приложения сосредоточенной нагрузки, в миллиметрах;

X – координата точки нахождения изгибающего момента, угла поворота и прогиба балки, в миллиметрах;

F – нагрузка, в ньютонах;

Ix – момент инерции сечения относительно оси, перпендикулярной действию нагрузки, в метрах 4;

Е – модуль упругости материала балки, в паскалях.

Расчет балки # 1.

1

Расчет изгибающего момента, угла поворота и прогиба в произвольно заданной точке консольно закрепленной балки под действием сосредоточенной нагрузки.

Граничные условия:

RL = 0 – реакция опоры в крайней левой точке;

ML = 0 – изгибающий момент в крайней левой точке;

θR = 0 – угол поворота в крайней правой точке;

YR = 0 – прогиб балки в крайней правой точке.

Расчет балки # 2.1

Расчет изгибающего момента, угла поворота и прогиба в произвольно заданной точке балки c защемленным концом и скользящей опорой под действием сосредоточенной нагрузки.

Граничные условия:

RL = 0 – реакция опоры в крайней левой точке;

θL = 0 – угол поворота в крайней левой точке;

θR = 0 – угол поворота в крайней правой точке;

YR = 0 – прогиб балки в крайней правой точке.

Расчет балки # 3.1

Расчет изгибающего момента, угла поворота и прогиба в произвольно заданной точке балки c защемленным концом и шарнирной опорой под действием сосредоточенной нагрузки.

Граничные условия:

МL = 0 – изгибающий момент в крайней левой точке;

YL = 0 – прогиб балки в крайней левой точке;

θR = 0 – угол поворота в крайней правой точке;

YR = 0 – прогиб балки в крайней правой точке.

Расчет балки # 4.1

Расчет изгибающего момента, угла поворота и прогиба в произвольно заданной точке балки c защемленными концами под действием сосредоточенной нагрузки.

Граничные условия:

θL = 0 – угол поворота в крайней левой точке;

YL = 0 – прогиб балки в крайней левой точке;

θR = 0 – угол поворота в крайней правой точке;

YR = 0 – прогиб балки в крайней правой точке.

Расчет балки # 5.1

Расчет изгибающего момента, угла поворота и прогиба в произвольно заданной точке балки c шарнирными опорами под действием сосредоточенной нагрузки.

Граничные условия:

МL = 0 – изгибающий момент в крайней левой точке;

YL = 0 – прогиб балки в крайней левой точке;

МR = 0 – изгибающий момент в крайней правой точке;

YR = 0 – прогиб балки в крайней правой точке.

Расчет балки # 6.1

Расчет изгибающего момента, угла поворота и прогиба в произвольно заданной точке балки c шарнирной и скользящей опорами под действием сосредоточенной нагрузки.

Граничные условия:

RL = 0 – реакция опоры в крайней левой точке;

θL = 0 – угол поворота балки в крайней левой точке;

МR = 0 – изгибающий момент в крайней правой точке;

YR = 0 – прогиб балки в крайней правой точке.

Расчет балки на прочность онлайн калькулятор

Балка длиной L загружена равномерно распределенной нагрузкой q либо сосредоточенной силой P, которые необходимо будет задать (как собрать нагрузки на балку можно получить тут Сбор нагрузок (калькулятор).

Все геометрические размеры сечения можно задать самому, поэтому в калькуляторе реализован огромный выбор самых различных балок: труба, швеллер, профильная труба, двутавр, уголок, пластина и др.
Расчет проходит по нормальным и касательным напряжениям, которые возникают из-за поперечной силы.
Касательные напряжения получаем по формуле Журавского и производим проверку с использованием главных напряжений по 3-ей теории прочности.
В онлайн расчет входят такие материалы, как сталь нескольких классов (С235, С245, С255, С345) и дерево трех сортов.

Для расчета вам необходимо:
1. Выбрать форму поперечного сечения (труба, швеллер, профильная труба, двутавр, уголок, пластина и др.)
2. Выбрать материал (сталь, дерево)
3. Выбрать необходимую расчетную схему
4. Выбрать вид нагрузки (распределенная по длине балки либо сосредоточенная)
5. Указать геометрические размеры, указанные на картинках
6. Задать нагрузку (нагрузку можно рассчитать онлайн здесь)


Также есть возможность выбора расчетной схемы: шарнир-шарнир, заделка-шарнир, заделка-заделка, свободный конец балки.
Коэффициенты поправки расчетного сопротивления дерева на изгиб приняты следующие:
Mдл = 0. 66 — совместное действие постоянной и кратковременной снеговой нагрузок
Mв = 0.9 — нормальные условия эксплуатации дерева (влажность менее 12%)
Mт = 0.8 — эксплуатация дерева при температуре 50 градусов
Mсс = 0.9 — срок эксплуатации конструкции 75 лет

При расчете уже учитывается собственный вес конструкции.


Последние изменения
1. Добавлена возможность расчета балки при сосредоточенной нагрузке
— Добавлена проверка устойчивости стенки и полки двутавра, швеллера, уголка, профильной трубы
— Исправлено расчетное сопротивление дерева на изгиб согласно СП 64.13330.2017 «Деревянные конструкции»
— Исправлены расчетные сопротивления стали
— Исправлено допустимое эквивалентное напряжение при действии нормальных и касательных напряжений
— Добавлена возможность поворота швеллера

Если данный калькулятор оказался Вам полезен – не забывайте делиться им с друзьями и коллегами ссылкой в соц. сети, а также посмотреть другие строительные калькуляторы онлайн, они простые, но здорово облегчают жизнь строителям и тем, кто решил сам строить свой дом с нуля.

Расчет балки на изгиб — Favorit-TK.ru

Рассчитывать балку на изгиб можно несколькими вариантами:

1. Расчет максимальной нагрузки, которую она выдержит
2. Подбор сечения этой балки
3. Расчет по максимальным допустимым напряжениям (для проверки)

Давайте рассмотрим общий принцип подбора сечения балки на двух опорах загруженной равномерно распределенной нагрузкой или сосредоточенной силой.

Для начала, вам необходимо будет найти точку (сечение), в которой будет максимальный момент. Это зависит от опирания балки или же ее заделки. Снизу приведены эпюры изгибающих моментов для схем, которые встречаются чаще всего.

После нахождения изгибающего момента мы должны найти момент сопротивления Wx этого сечения по формуле приведенной в таблице:

Далее, при делении максимального изгибающего момента на момент сопротивления в данном сечении, мы получаем максимальное напряжение в балке и это напряжение мы должны сравнить с напряжением, которое вообще сможет выдержать наша балка из заданного материала.

Для пластичных материалов (сталь, алюминий и т.п.) максимальное напряжение будет равно пределу текучести материала, а для хрупких (чугун) – пределу прочности. Предел текучести и предел прочности мы можем найти по таблицам ниже.

Давайте рассмотрим пару примеров:

1. Вы хотите проверить, выдержит ли вас двутавр №10 (сталь Ст3сп5) длиной 2 метра жестко заделанного в стену, если вы на нем повисните. Ваша масса пусть будет 90 кг.

Для начала нам необходимо выбрать расчетную схему.

На данной схеме видно, что максимальный момент будет в заделке, а поскольку наш двутавр имеет одинаковое сечение по всей длине, то и максимальное напряжение будет в заделке.

Давайте найдем его:
P = m * g = 90 * 10 = 900 Н = 0.9 кН
М = P * l = 0.9 кН * 2 м = 1.8 кН*м

По таблице сортамента двутавров находим момент сопротивления двутавра №10.
Он будет равен 39.7 см3. Переведем в кубические метры и получим 0.0000397 м3.
Далее по формуле находим максимальные напряжения, которые у нас возникают в балке.
б = М / W = 1.8 кН/м / 0.0000397 м3 = 45340 кН/м2 = 45.34 МПа

После того, как мы нашли максимальное напряжение, которое возникает в балке, то мы его может сравнить с максимально допустимым напряжением равным пределу текучести стали Ст3сп5 – 245 МПа.
45.34 МПа < 245 МПа – верно, значит данный двутавр выдержит массу 90 кг.

2. Поскольку у нас получился довольно-таки большой запас, то решим вторую задачу, в которой найдем максимально возможную массу, которую выдержит все тот же двутавр №10 длиной 2 метра.

Если мы хотим найти максимальную массу, то значения предела текучести и напряжения, которое будет возникать в балке, мы должны приравнять (б=245 Мпа = 245 000 кН*м2).

Далее по формуле б = М / W, находим максимальный момент.
М = б * W = 245 000 * 0.0000397 = 9.73 кН * м

Тогда по формуле M = P * L найдем P:
P = 9,73 кН/м / 2м = 4,87 кН = 487 кг

Итак, максимальная масса, которую выдержит двутавр №10 – 487 кг. Число это грубое, поскольку для простоты расчета мы не учитывали различные коэффициенты запаса, поэтому, чтобы подстраховаться, возьмите некий двукратный запас по прочности.

Полный расчет балки на прочность и жесткость. Решение задачи

Задача

Произвести полный расчет на прочность и проверить жесткость статически определимой двутавровой двухопорной балки (рис. 1) при следующих данных: F=40кН, q=30 кН/м, a=0,8 м, l=4м, допустимые нормальные и касательные напряжения: [σ]=160 МПа и [τ]=100 МПа, допустимый прогиб балки [f]=l/400

Рис. 1

Решение

Подготовка расчетной схемы к решению задачи:

Определение опорных реакций

Подробно, пример определения опорных реакций для балки рассмотрен здесь

А также в нашем коротком видеоуроке:

Из Σmв=0

Из ΣmА=0

Построение эпюр Q и М

Подробный пример построения эпюр поперечных сил Q и изгибающих моментов M для балки

Видео про расчет значений Q и M для построения эпюр:

В пролете балки 0 ≤ z2 ≤ l

QII= — RB+ qz2= -52+30∙z2
QII(z=0)= -52 кН
QII(z=l)= -52+30∙4=68 кН

MII=RB∙z2-qz22/2=52z2-30∙z22/2
MII (z=0)= 0
MII (z=l)= -32 кНм

На консоли l ≤ z1≤ (l+a)

QI= — RB+ ql — RA=-52+30∙4-108=-40 кН

MI=RB z1-ql(z1-l/2)+RA(z1-l)=52z1-30∙4(z1-4/2)+108(z1-4)
MI (z=l)= -32 кНм
MI (z=l+a)= 0

По этим данным построены эпюры Q и М.

Короткое видео о том, как надо строить эпюры:

Так как Мmах = 45 кНм, то

Wx≥Mmax / [σ] = 45∙103 / 160∙106= 0,281 м3= 281 см3.

О том, как подбирается сечение балки

По сортаменту выбираем двутавр № 24, для которого Wx = 289 см3, Ix= 3460 см4, Smax = 163 см3, h = 24 см, bп = 11,5 см, t = 0,95 см, d = bc = 0,56 см, h0 = h-2t = 22,1 см.

Этот двутавр будет работать при максимальном нормальном напряжении в крайнем волокне опасного сечения.

σmax = Mmax / Wx = 45∙103 / 289∙10-6= 156∙106 Па = 156 МПа

Проверка сечения балки по касательным напряжениям

Так как Qmax = 68 кН, то

Построение эпюр нормальных σ и касательных τ напряжений в неблагоприятном сечении балки:

Построение эпюры нормальных напряжений

Построение эпюры касательных напряжений

В отношении главных напряжений неблагоприятным является сечение над левой опорой, в котором:

М = -32 кНм и Q = 68 кН.

Значение напряжений в различных точках по высоте двутавра сведены в таблицу 1

Таблица 1

Результаты расчета в примере

Проверка прочности балки по главным напряжениям

Наиболее опасной точкой в неблагоприятном сечении является точка 3. В этой точке σ1=118 МПа и σ3= -16 МПа. Проверяем прочность в этой точке по третьей гипотезе прочности согласно неравенству σ1 — σ3≤ [σ].

Так как 118 — ( -16) = 134 < 160, то выбранное сечение удовлетворяет условию прочности и по главным напряжениям.

Расчет перемещений сечений (прогибов балки)

Универсальные уравнения МНП для сечения z:

Опорные условия:

1) при z=0: y(z)=0, следовательно, y0=0

2) при z=l: y(z)=0 находим θ0

откуда θ0= -8,48∙10-3 радиан.

Прогиб в пролете при z=l/2=4/2=2 м.

Аналогично определяется прогиб на конце консоли при z = l + a =4+0,8 = 4,8 м.

Проверка жесткости балки

— пролетной части:

yc=0,98 см < 1/400 = 400/400 = 1 см

— консольной части:

yD=0,33 см < 2a/400 = 2∙80/400 = 0,4 см.

Следовательно, принятая двутавровая балка удовлетворяет требуемому условию жесткости.

Другие примеры решения задач >

Расчет балки онлайн — Калькулятор балок перекрытия из дерева —  

Калькулятор подбора деревянных двутавровых балок

SIA I-beams производит износоустойчивые деревянные двутавры. Такие балки показали себя как незаменимый стройматериал при строительстве зданий в Северной Америке, понемногу они начинают завоевывать и рынки Европы.

Чтобы правильно произвести расчет необходимого количества балок, мы создали расчетный калькулятор, который вам поможет быстро и удобно рассчитать шаг между балками и их тип в зависимости от расстояния между стенами и от нагрузок в конкретном случае.

Как пользоваться калькулятором:

  1. Вводим расчетную длину пролета. Для балок перекрытия — это наибольший пролет, т.е. наибольшее расстояние между соседними стенами, на которые опирается балка. Для стропил кровли – это горизонтальное расстояние (проекция мест опоры, обычно расстояние между осями) между местами опора балки (сама балка длиннее, чем эта проекция, т.е. чем больше угол, тем длиннее балка).
  2. Для стропил кровли вводим угол наклона. Угол наклона – наклон стропил к горизонтали.
  3. Вводим шаг – это межцентровое расстояние между соседними балками.
  4. 4. Можно изменить постоянную нагрузку. В соответствии с нормативом EN 1991, постоянную нагрузку рассчитывают по плотности конструкции пола/перекрытия/крыши, помноженной на коэффициент надежности. Согласно EN 1990, коэффициент надежности для постоянных нагрузок — 1,35, а для временных — 1,5.
  5. Можно изменить временную нагрузку. В соответствии с нормативом EN 1991, величины временной нагрузки принимаются в зависимости от предполагаемого использования перекрытия. Для перекрытий жилых помещений можно принимать временную нагрузку 200 kg/m2. При расчете стропильной системы нагрузки от снега принимаются согласно LBN-003-1, таблица 16.2. Для Риги это равняется 125 kg/m2.

    *В расчетном калькуляторе включено определение расчетной нагрузки при соответствующих коэффициентах надежности: согласно EN 1990 для постоянных нагрузок это — 1,35 а для временных нагрузок — 1,5. В калькулятор вводятся нагрузки без учета коэффициентов надежности. – это повторение из п.4.

    *Величина используемой расчетной нагрузки будет индивидуальной — в зависимости от конкретной ситуации.

  6. Когда все упомянутые данные введены в таблицу, можно ознакомиться с результатом. Внизу находится табличка с имеющимися в нашем ассортименте балками. Зеленым цветом закрашены все балки, которые можно использовать, а красным – несущая способность которых не соответствует заданным вами параметрам. Чтобы изменить результат, советуем изменить шаг балок.

Калькуляторы расчета деревянных балок — Доктор Лом

Ну а теперь поговорим о положительных качествах калькуляторов:

1. Все калькуляторы выполняют расчет согласно требований СП 64.13330.2011. Есть все необходимые данные для построения эпюр поперечных сил, изгибающих моментов, углов поворота и прогибов, а также нормальных сил, если это требуется.

2. Калькуляторы прекрасно подходят как для домохозяек, впервые задумавшихся о расчете конструкций, так и для продвинутых пользователей, понимающих толк в сопромате. Для тех и других есть первая вкладка, где вводятся данные — длина пролета, значение нагрузки (и другие, если требуется). Калькулятор тут же, в этой же вкладке, выдает ближайшее сечение деревянной балки, удовлетворяющее условиям по прочности, показывает максимальный прогиб балки в сантиметрах и отношение прогиба к длине пролета и проходит ли это сечение по общим требованиям по прогибу.

3. Также калькулятор показывает, проходит ли данное сечение деревянной балки по нормальным напряжениям на опорных участках (подобной опции в on-line калькуляторах я пока не встречал). Проверить, проходит сечение или нет по скалывающим напряжениям, можно в соответствующей вкладке, но как правило если сечение проходит по прочности, то и по скалывающим напряжениям тоже проходит.

4. Предлагаемое калькулятором сечение далеко не всегда есть в свободном доступе, поэтому на первой вкладке есть возможность проверить прочность балки из имеющегося спектра (того, что есть на ближайшем складе пиломатериалов, ну или на складе, где пиломатериалы стоят дешевле всего — это уже вам решать). Для этого достаточно ввести ширину и высоту деревянной балки (в сантиметрах). Калькулятор определит, можно или нет использовать балку такого сечения, исходя из требований прочности, и покажет, какой прогиб будет иметь такая балка и проходит ли такая балка по нормальным напряжениям на опорных участках и по общим требованиям по прогибу.

5. Кроме вышеуказанного калькулятор покажет, сколько будет весить деревянная балка, что бывает весьма полезно, если вы планируете укладывать эти балки самостоятельно.

6. Функция примерной цены балки работает следующим образом, в основу расчета заложены брусья длиной 6, 3 и 2 м, например если ваша балка длиной 5 метров, то все равно вы будете платить за 6 метров и у вас будет 1 м отходов. Если у вас есть возможность заказать балки нужного размера без отходов, то не обращайте на данную опцию внимания.

7. Для продвинутых пользователей (как впрочем и для обычных) есть возможность указать расчетное сопротивление древесины, модули упругости древесины и материала опоры, отличное от тех, что даются по умолчанию. Это не очень сильно повлияет на результаты расчетов, но все-таки.

8. Конечно же есть возможность заглянуть в другие вкладки и проверить точность расчетов. А кроме того вы можете изменить калькулятор под свои нужды (если соображаете в экселе), что иногда бывает также весьма полезно.

Для балок из LVL бруса все данные только в соответствующей вкладке, на первую вкладку выносить ничего не стал, чтобы не усложнять восприятие. Да и потребности в расчете таких балок возникают далеко не у многих. Тем не менее, если вам известны все необходимые параметры LVL бруса, то вы можете внести их на первой вкладке и посмотреть результат.

Ну а теперь непосредственно ссылки на сами калькуляторы.

1. Калькулятор для расчета балок на действие равномерно распределенной нагрузки. Такая нагрузка — одна из самых распространенных, соответственно и такой калькулятор будет одним из самых востребованных. Во всяком случае мне так кажется.

2. Калькулятор для расчета балок на действие сосредоточенной нагрузки. Этот калькулятор больше для студентов, но и простым людям может пригодиться.

3. Калькулятор для расчета балок на действие наклонной равномерно распределенной нагрузки. Этот калькулятор может использоваться при расчете стропил или других наклонных элементов конструкции.

4. Калькулятор для расчета балок на действие равномерно распределенной нагрузки, действующей не по всей длине пролета балки. Этот калькулятор тоже по большей части для людей, изучающих теорию сопротивления материалов.

Скорее всего со временем появятся и другие калькуляторы.

Онлайн калькулятор балок перекрытия из дерева.

Как сделать расчет лаг для пола

Еще не так давно казалось, что ламинатные покрытия полов полностью вытеснят всем привычные деревянные. Их относительная дешевизна привлекала многих застройщиков. Так продолжалось до тех пор, пока большинство из них не поняли, что дешевизна материала вполне отвечает его «дешевым» эксплуатационным характеристикам. Теперь многие желают иметь у себя настоящее деревянное покрытие из натуральных материалов. Как правильно уложить доски на лаги?

Что такое лаги и какие у них преимущества

Лаги – мощные поперечные балки, служащие основанием для укладки досок, чаще всего изготавливаются из дерева. Это могут быть распиленные квадратные или прямоугольные брусья различного размера. Использование лаг позволяет:

  • Улучшить шумоизоляцию между верхним и нижним помещениями. Одновременно улучшаются эксплуатационные показатели по теплопроводности.
  • Сделать нагрузку на несущее межэтажное перекрытие более равномерной – исключается появление трещин на потолке.
  • В свободное пространство между лагами и половым покрытием прячется большинство инженерных коммуникаций.
  • Существенно облегчается проведение ремонтных работ в случае необходимости. Ремонт выполняется быстро, все материалы полностью пригодны к повторному использованию – ощутимо снижается стоимость выполнения работ.

Выбор древесины для лаг и расчет их сечения

Для лаг можно использовать недорогие сорта хвойных пород древесины, и т.п.. Влажность конструкций должна быть в пределах 18÷20%, желательно перед укладкой материал несколько дней выдержать в комнатных условиях. За это время они «отрегулируют» свою влажность, что исключит чрезмерные колебания линейных размеров во время изменения влажности.

Сечение бруса лучше делать прямоугольным, отношение сторон 1×2. Это существенно уменьшает кубатуру материала и общую сметную и при этом почти не влияет на несущие характеристики. Конкретные размеры нужно подбирать с учетом ширины пролета между ними и расчетной максимальной нагрузкой на пол. В таблице даны примерные сечения лаг для различных размеров комнат. Принимается во внимание, что расстояние между лагами составляет 0,7 метра.

Если размеры помещения не совпадают с данными в таблице – лучше выбирать сечение лаг «с запасом».

Расстояние между лагами следует согласовывать с толщиной настилаемых досок. В таблице приводятся параметры толщины половых досок с учетом шага лаг.

Установка лаг

Лаги в зависимости от конструкционных особенностей здания могут устанавливаться непосредственно по бетонному перекрытию, на деревянные балки перекрытия или на кирпичные столбики.

Чаще всего лаги устанавливаются на бетонные перекрытия. Во время установки нужно выполнять три условия:

  • Обязательно прокладывать слой надежной гидроизоляции между бетоном и деревянными конструкциями. Это предотвратит их от быстрого разрушения вследствие повышения влажности.
  • Все лаги должны находиться на одном уровне. Достигается это условие при помощи водяного уровня и обыкновенной нити. По уровню делается «нулевая» разметка на противоположных стенах помещения, выставляются крайние лаги. Между ними натягивается нить и по ее уровню укладываются все остальные лаги с соблюдением необходимых расстояний.
  • Лаги должны быть надежно зафиксированы на упорах. Во время настила пола они не должны изменять своего пространственного положения.

При желании между лагами и половыми досками можно уложить слой теплоизоляции. Это может быть и минеральная вата, и листы пенопласта, и слой керамзита. Выбор утеплителя зависит от материального положения и личных предпочтений владельца квартиры.

Установка лаг на грунт

Такой метод применяется во время строительства дачных или загородных домов. Перед началом выполнения работ необходимо удалить плодородный слой грунта, желательно насыпать шар песка или гравия. После этого производится разметка, определяются места установки кирпичных или бетонных столбиков. Столбики выкладываются по уровню, выравнивание по высоте лучше производить цементно-песчанной смесью. Использовать для этих целей различные деревянные прокладки довольно рискованно – они могут потерять свои несущие способности, что вызовет нарушение жесткости всей конструкции. Пол начнет «скрипеть», нарушится его прямолинейность.

Первые столбики должны быть на удалении от стены не более чем на 20 см, лучше сделать больше рядов столбиков, чем рисковать устойчивостью всего полового покрытия. Помните, что кирпичная кладка не может быть в земле, она всегда должна ложиться на бетонное основание и только с использованием слоя гидроизоляции.

Укладка досок

Довольно трудоемкий процесс, требует определенных практических навыков работы. Первая укладывается с противоположной к входным дверям стены, не забывайте по периметру делать зазор в 1÷2 см для компенсации явлений расширения. Первую доску не спешите фиксировать, несколько раз проверьте ее параллельность как к ближней, так и к противоположной стене, эти щели потом закроются плинтусами. После фиксации первого ряда начинайте укладывать последующие ряды.

Каждый ряд досок прижимается к предыдущему. Для этого применяются специальные приспособления: металлические П-образные скобы, деревянные прокладки и клинья. Скобы прибиваются к лагам, при помощи прокладок и клиньев доски плотно подгоняются друг к другу и фиксируются в таком положении. Бывают случаи, когда половая доска имеет большие искривления. Тогда придется чаше использовать клинья, прижимать доску до тех пор, пока полностью не исчезнут зазоры.

Финишные работы

Половое покрытие из натуральных досок придется отшлифовать электрическими шлифовальными машинами. Это не только окончательно выровняет плоскость пола, но и подготовит под покрытие лаком или красками. После шлифовки необходимо тщательно убрать все опилки и можно прибивать по периметру плинтуса. Стоит заметить, что сегодня стоимость натурально пола не каждому по карману. Но его высокая цена полностью оправдывается отличными эксплуатационными характеристиками.

Деревянный дом и баня – мечта многих горожан. Каждый из тех, кому приходилось своими руками возводить сруб из дерева, отлично знает назначение лаг и балок.
Это несущие конструкции здания, поэтому подбирать материал для них, а также их количество нужно очень тщательно. Лаги для пола рекомендуется делать из сухого первосортного материала, обработанного антисептическим и огнезащитным составами. Наиболее распространенный способ их установки – монтаж по балкам, врезанным в стены в процессе строительства.

Расчёты лаг для пола делается, с учетом основных параметров, лаги должны быть в 1,5-2 раза выше высоты настила, иначе гвозди не смогут прочно удерживать доски пола.

Устанавливаются они с учетом того, что между ними обычно размещают материал утеплителя. Это может быть керамзит, пенопласт, но чаще всего используется минеральная вата, спрессованная в плиты шириной 50-60 см. На таком же расстоянии друг от друга монтируются лаги. На них стелется деревянный пол, который при правильном монтаже прослужит несколько десятков лет. Лаги бывают изготовлены из брусков, балок, досок. Они служат для перераспределения нагрузок от пола, а также являются фиксатором, объединяющим все узлы и детали в цельную систему.

Преимущества использования лаг

Пол на лагах обладает определенной степенью функциональности. В пространстве между ними можно проложить трубы, провода, теплоизоляционные материалы.

Бруски стоят относительно недорого. Установка их доступна каждому.

Эти опоры для пола способны выдерживать нагрузку в 5 т на каждый квадратный метр.

Рисунок 1. Схема крепление деревянных балок.

При ремонте пола часто достаточно бывает отремонтировать лагу. Перестилать пол при этом не требуется.

Конструкция не имеет большой массы. Нагрузка на перекрытие оказывается гораздо меньше, чем при цементной стяжке.

Заполненное минеральной ватой пространство сохранит тепло в доме и защитит его от лишнего шума.

Бруски позволяют вывести плоскость пола на любую высоту.

Уложенные на место конструкции не требуют проведения дополнительных работ. Можно сразу настилать покрытие пола.

Недостатки пола на лагах:

  1. Комната теряет несколько сантиметров высоты.
  2. Высокая трудоемкость. Требуется тщательно разметить и выровнять все элементы конструкции.

Расчет некоторых параметров лаг

Расчет лаг для пола производят, учитывая основные параметры. Лаги для пола должны быть в 1,5-2 раза выше высоты напольного настила, иначе гвоздь не в состоянии будет прочно удерживать доски пола. Если толщина половой доски составляет 50 мм, то высота брусков должна быть около 100 мм. Если черновой пол делается из фанеры или иного листового материала, имеющего толщину 20 мм, брусья могут быть значительно ниже, 30-40 мм.

Материал для изготовления деревянных лаг следует выбирать хвойных пород. Влажность заготовок не должна превышать 20%. Сечение брусков выбирается прямоугольное. Их можно выпилить из доски толщиной 50-60 мм. Укладывают готовые изделия поперек света, исходящего от окон. Шаг укладки – от 40 до 70 см. Зная шаг укладки и размеры помещения, нетрудно произвести расчет необходимого количества элементов. Перед монтажом все деревянные элементы дважды обрабатываются антисептическим составом. Антисептик можно заменить обычным горячим битумом.

Рисунок 2. Регулировочные втулки. Применяются для выравнивайте полов на лагах.

На практике очень часто высота лаг выбирается с учетом толщины слоя утеплителя. В качестве утеплителя для пола обычно используют минеральную вату, выпускаемую плитами, толщина которых составляет 50 мм. Такой же высоты должны быть и лаги для пола. Если решено укладывать теплоизоляцию двойным слоем, то бруски нужны высотой 100 мм. Расстояние между ними зависит от толщины материала чернового пола. Чем черновой настил тоньше, тем чаще устанавливаются лаги. При толщине фанеры, которая может использоваться в качестве подложки под чистовой настил, в 12 мм размер промежутка между брусками составляет 30 см.

Чаще всего черновой пол выполняют из шпунтованной доски. Доски должны быть еловыми, сосновыми или пихтовыми. Для чистового пола они не подходят, так как древесина очень мягкая, на ней остаются даже следы от тонких каблуков. Сверху нужно обязательно укладывать ламинат или иное финишное покрытие. Толщина досок при нормальном шаге лаг в 50 см рекомендуется не менее 35 мм. В большинстве случаев расчет шага брусков производится с учетом толщины материала пола:

Рисунок 3. Крепеж. Применяется для крепление деревянных конструкций.

Толщина доски (мм) – шаг лаг (мм):

  • 20 – 300;
  • 24 – 400;
  • 30 – 500;
  • 35 – 600;
  • 40 – 700;
  • 45 – 800;
  • 50 – 1000.

Для изготовления этих деталей используется не только древесина, но и железобетон, различные полимеры и металлы. Железобетонные изделия отличаются высокой прочностью. Их можно использовать при строительстве дома за городом. Остальные материалы можно применять при проведении ремонта полов.

Если основой пола являются деревянные балки, лаги можно устанавливать, прикрепляя их сбоку с помощью саморезов (рис. 1). Размер крепежей должен превышать толщину бруска в 2,5 раза при диаметре 6 мм. Положительный момент этого способа состоит в том, что при регулировке высоты отдельных лаг не требуется применения дополнительных регулировочных подкладок.

В строительстве иногда используются особые деревянные или пластиковые изделия, имеющие отверстия, в которые вставляются небольшие регулировочные втулки из пластмассы. Они способствуют быстрому выравниванию поверхности, образуемой лагами. Такие изделия монтируются очень быстро и не требуют применения подкладок (рис. 2).

Рисунок 4. Схема монтажа пола на лагах.

Деревянные элементы перед установкой необходимо защитить от различных микроорганизмов и вредителей-древоточцев, обработав материал дезинфицирующим, затем водоотталкивающим составом.

В помещениях с низкими потолками лучше использовать другие методы монтажа пола. При выполнении расчета нельзя забывать, что бруски уменьшают размер комнаты по высоте на 10 см и более.

Половицы или листы чернового настила следует крепить к каждой лаге.

Торцы конструктивных элементов не должны касаться стен здания. Между ними должен оставаться зазор не менее 5 см.

Вместо определенного сечения деревянного бруса можно использовать доски, попарно соединенные друг с другом и достигающие размеров нужного бруса в поперечнике. Несколько большие размеры не возбраняются. Доски устанавливаются на ребро.

К бетонной основе лаги можно крепить специальными оцинкованными металлическими уголками, которые фиксируются на основании с помощью дюбелей и саморезов. Вместо уголков часто используются П-образные приспособления (рис. 3).

При необходимости бруски, из которых выполнены лаги, стыкуются друг с другом для достижения нужной длины. Под местом стыка обязательно должна быть прочная опора. Этой опорой часто служит кирпичный столб. Под его сооружение нужно выкопать ямку глубиной около 10 см. Ее засыпают песком и обильно поливают водой. Сверху песчаную подушку накрывают слоем полиэтилена. На него кладут цементно-песчаный раствор и выкладывают столбик из красного кирпича.

Можно выложить столбики рядами и на них закрепить лаги (рис. 4). Размер столбиков – 25х25 см. Расчет количества кирпичей не представляет сложностей.

Конструкция пола на лагах позволяет сразу смонтировать черновой и чистовой настил.

Лаги можно делать из различных материалов. Чаще всего на их изготовление идет деревянный брусок или доска. А материалов для финишной отделки существует очень много. Выбор их зависит только от предпочтений хозяев и содержимого их кошелька. Сделать расчет материалов для изготовления лаг для пола поможет выполненный на бумаге план комнат с точно указанными размерами. Все расчеты лучше делать до начала выполнения основных монтажных работ.

Тематика этой статьи – расчет лаг для пола и их укладка своими руками. Мы выясним, из чего и какого сечения делаются лаги, с каким шагом их лучше укладывать при разных типах настила, как дезинфицировать для предотвращения загнивания и как монтировать на основаниях разных типов.

Зачем это нужно


Простой ответ на этот вопрос, вероятно, может дать даже школьник младших классов. Чтобы не класть доски на неровный пол, верно?

Да, но этот ответ, мягко говоря, неполон.

У лаг под настилом много других функций.

  • Они обеспечивают полноценную вентиляцию нижней стороны деревянного настила, предотвращают его загнивание.

Обратите внимание: этот пункт особенно важен при укладке пола по грунту. Если у вас высокий уровень грунтовых вод, сырость может создавать серьезные проблемы даже при высоком подполе.

  • Лаги улучшают шумоизоляцию, обеспечивая между настилом и основанием буферное пространство.
  • Место под настилом часто используется для укладки утеплителя и для прокладки инженерных коммуникаций.

Между лаг уложен утеплитель – стекловата.

  • Наконец, при неровном основании в целом куда более прочным, создавая точки опоры для настила с постоянным небольшим шагом.

Применяемые материалы


В теории лаги могут быть изготовлены из любого материала достаточной прочности и постоянных линейных размеров – из металла, пластика, компаунда на основе синтетических смол и целлюлозы. Однако цена этих материалов делает их неконкурентоспособными на фоне древесины. Как правило, используется обычный брусок.

Какие породы допустимо применять? Оптимальный вариант – устойчивая к гниению и прочная лиственница, однако куда чаще используются дешевые ель, сосна и пихта. Смоляные кармашки и продольные трещины в этом случае не сказываются на функциональности бруса, поэтому можно смело брать пиломатериалы 2-3 сорта.

Единственное, на чем не стоит экономить – влажность бруска. Брус так называемой атмосферной влажности гарантированно будет деформирован при сушке
.

Влажность материала для лаг не должна превышать 20%. В первую очередь это касается случаев, когда лаги укладываются на столбики по грунту, без надежной фиксации.

Приведем значения шага для настила из доски различной толщины.

  • Если планируемая толщина настила равна 20 миллиметрам, максимальный шаг не превышает 30 сантиметров.
  • Для 25-миллиметровой доски он равен уже 40 см.
  • 30 мм – 50 см.
  • 35 – 60.
  • 40 – 70.
  • 45 – 80.
  • 50 – 100 сантиметров.

Для фанеры или ОСП инструкция несколько иная.

Эти материалы обладают большей жесткостью на изгиб, но производятся в меньшем диапазоне толщины.

  • При толщине настила в 15 – 18 миллиметров можно ориентироваться на шаг в 40 сантиметров.
  • При толщине 22 – 24 мм его допустимо увеличить до 60 см.

Для расчета материала, в общем-то, несложно найти в сети калькулятор лаг пола. При заданной длине пролета он позволит вычислить необходимое сечение бруса из произвольной породы древесины и укажет максимально допустимую нагрузку.

Монтаж


Как укладывать лаги на разных основаниях?

Бетон

  1. Если помещение находится на первом этаже, перекрытие гидроизолируется полиэтиленом.

Совет: если использовать фольгоизол – вспененный полиэтилен с фольгированным слоем – он заодно ощутимо уменьшит потери тепла за счет излучения. Он укладывается фольгой вверх, в сторону помещения с большей температурой.

  1. Брус раскладывается с заданным шагом; затем он выставляется по уровню с помощью подкладок – отрезков бруска и фанеры. В горизонт выставляется не только отдельный брус, но и соседние лаги. Любые три бруска должны быть расположены на одной линии (это несложно проверить правилом).
  2. Затем брус фиксируется к полу. Лучше – анкерами под отвертку: в этом случае отверстия можно сверлить прямо через дерево, а гайки не придется топить, высверливая отверстия значительной глубины и ослабляя материал.

На фото – альтернативный вариант. К перекрытию крепятся подставки, лаги притягиваются к ним саморезами.

Деревянные балки

При креплении бруска поперек балок, он притягивается к ним обычными саморезами достаточной длины – тоже, разумеется, с подкладками, позволяющими вывести пол в горизонт. В этом случае предварительная обработка лаг пола, помимо пропитки антисептиком, включает обязательное засверливание отверстий – иначе брусок легко расколоть.

Если брус крепится вдоль лаг для компенсации их разброса по высоте, его можно не только уложить сверху, но и подшить к ним сбоку. В этом случае пол будет выровнен с меньшей потерей высоты помещения, а сами лаги могут быть заметно уже.

Грунт

Вкратце укладку по столбам мы уже затронули.

Основные этапы выглядят так:

  1. Под каждый столб роется яма глубиной от 10 см. Она засыпается песком и проливается водой для лучшей усадки.
  2. Песок перекрывается полиэтиленом. Затем на лепешке раствора сооружается столбик размером 25х25 см (длиной и шириной в кирпич).
  3. Выведенные раствором в уровень столбики перекрываются рубероидом; просушенный до 16-18 % влажности брус укладывается на них без фиксации. Лаги краями укладываются на ростверк, один из нижних венцов сруба или подшиваются к стенам оцинкованными уголками.

монтажа половых лаг. На их оптимальные размеры и шаг тоже есть разные взгляды. Одну из альтернативных точек зрения вы найдете в видео в этой статье. Успехов!

Расчет деревянной балки Онлайн, расчет несущей способности и прогиба деревянных балок

Распределенная нагрузка (перекрытия)

Шаг балок,мм

Нагрузка по площади, кг/кв.м

Распределенная нагрузка, кг/кв.м 150

При относительном прогибе
1/2501/2001/150

максимально допустимый прогиб для междуэтажных перекрытий, мм 16

Расчетный прогиб, мм 12

Расчетный относительный прогиб 1/333

Запас по прогибу в 1.33 раза

Разрушающая нагрузка, кг 2475

Сосредоточенная нагрузка (ригели)

Сосредоточенная нагрузка, кг

Расчетный прогиб, мм 16

Запас по прогибу в 1.33 раза

Разрушающая нагрузка, кг 1238

Расчет несущей способности и прогиба деревянных балок

Чтобы построить деревянный дом необходимо провести расчёт несущей способности деревянной балки. Также особое значение в строительной терминологии имеет определение  прогиба.

Без качественного математического анализа всех параметров просто невозможно построить дом из бруса. Именно поэтому перед тем как начать строительство крайне важно правильно рассчитать прогиб деревянных балок. Данные расчёты послужат залогом вашей уверенности в качестве и надёжности постройки.

Что нужно для того чтобы сделать правильный расчёт

Расчёт несущей способности и прогиба деревянных балок не такая простая задача, как может показаться на первый взгляд. Чтобы определить, сколько досок вам нужно, а также, какой у них должен быть размер необходимо потратить немало времени, или же вы просто можете воспользоваться нашим калькулятором.

Во-первых, нужно замерить пролёт, который вы собираетесь перекрыть деревянными балками. Во-вторых, уделите повышенное внимание методу крепления. Крайне важно, насколько глубоко фиксирующие элементы будут заходить в стену. Только после этого вы сможете сделать расчёт несущей способности вместе с прогибом и ряда других не менее важных параметров.

Длина

Перед тем как рассчитать несущую способность и прогиб, нужно узнать длину каждой деревянной доски. Данный параметр определяется длиной пролёта. Тем не менее это не всё. Вы должны провести расчёт с некоторым запасом.

Важно! Если деревянные балки заделываться в стены — это напрямую влияет на их длину и все дальнейшие расчёты.

При подсчёте особое значение имеет материал, из которого сделан дом. Если это кирпич, доски будут монтироваться внутрь гнёзд. Приблизительная глубина около 100—150 мм.

Когда речь идёт о деревянных постройках параметры согласно СНиПам сильно меняются. Теперь достаточно глубины в 70—90 мм. Естественно, что из-за этого  также изменится конечная несущая способность.

Если в процессе монтажа применяются хомуты или кронштейны, то длина брёвен или досок соответствует проёму. Проще говоря, высчитайте расстояние от стены до стены и в итоге сможете узнать несущую способность всей конструкции.

Важно! При формировании ската крыши брёвна выносятся за стены на 30—50 сантиметров. Это нужно учесть при подсчёте способности конструкции противостоять нагрузкам.

К сожалению, далеко не всё зависит от фантазии архитектора, когда дело касается исключительно математики. Для обрезной доски максимальная длина шесть метров. В противном случае несущая способность уменьшается, а прогиб становится больше.

Само собой, что сейчас не редкость дома, у которых пролёт достигает 10—12 метров. В таком случае используется клееный брус. Он может быть двутавровым или же прямоугольным. Также для большей надёжности можно использовать опоры. В их качестве идеально подходят дополнительные стены или колоны.

Совет! Многие строители при необходимости перекрыть длинный пролёт используют фермы.

Общая информация по методологии расчёта

В большинстве случаев в малоэтажном строительстве применяются однопролётные балки. Они могут быть в виде брёвен, досок или брусьев. Длина элементов может варьироваться в большом диапазоне. В большинстве случаев она напрямую зависит от параметров строения, которые вы собираетесь возвести.

Внимание! Представленный в конце странички калькулятор расчета балок на прогиб позволит вам просчитать все значения с минимальными затратами времени. Чтобы воспользоваться программой, достаточно ввести базовые данные.

Роль несущих элементов в конструкции выполняют деревянные бруски, высота сечения которых составляет от 140 до 250 мм, толщина лежит в диапазоне 55—155 мм. Это наиболее часто используемые параметры при расчёте несущей способности деревянных балок.

Очень часто профессиональные строители для того чтобы усилить конструкцию используют перекрёстную схему монтажа балок. Именно эта методика даёт наилучший результат при минимальных затратах времени и материалов.

Если рассматривать длину оптимального пролёта при расчёте несущей способности деревянных балок, то лучше всего ограничить фантазию архитектора в диапазоне от двух с половиной до четырёх метров.

Внимание! Лучшим сечением для деревянных балок считается площадь, у которой высота и ширина соотносятся как 1,5 к 1.

Как рассчитать несущую способность и прогиб

Стоит признать, что за множество лет практики в строительном ремесле был выработан некий канон, который чаще всего используют для того, чтобы провести расчёт несущей способности:

M/W<=Rд

Расшифруем значение каждой переменной в формуле:

  • Буква М вначале формулы указывает на изгибающий момент. Он исчисляется в кгс*м.
  • W обозначает момент сопротивления. Единицы измерения см3.

Расчёт прогиба деревянной балки является частью, представленной выше формулы. Буква М указывает нам на данный показатель. Чтобы узнать параметр применяется следующая формула:

M=(ql2)/8

В формуле расчёта прогиба есть всего две переменных, но именно они в наибольшей степени определяют, какой в конечном итоге будет несущая способность деревянной балки:

  • Символ q показывает нагрузку, которую способна выдержать доска.
  • В свою очередь буква l — это длина одной деревянной балки.

Внимание! Результат расчёт несущей способности и прогиба зависит от материала из которого сделана балка, а также от способа его обработки.

Насколько важно правильно рассчитать прогиб

Этот параметр крайне важен для прочности всей конструкции. Дело в том, что одной стойкости бруса недостаточно для долгой и надёжной службы, ведь со временем его прогиб под нагрузкой может увеличиваться.

Прогиб не просто портит эстетичный вид перекрытия. Если данный параметр превысит показатель в 1/250 от общей длины элемента перекрытия, то вероятность возникновения аварийной ситуации возрастёт в десятки раз.

Так зачем нужен калькулятор

Представленный ниже калькулятор позволит вам моментально просчитать прогиб, несущую способность и многие другие параметры без использования формул и подсчётов. Всего несколько секунд и данные по вашему будущему дому будут готовы.

Расчет несущей способности балок перекрытия и несущих конструкций в Москве

Нюансы расчета балок перекрытия

Балочные системы получили наибольшее распространение, изготавливаются из стали, бетона, дерева. Стальные швеллера предназначены для высокопрочных построек, бетонные отличаются простотой монтажа и небольшой теплопроводностью, деревянные максимально доступны по цене. Ведущие технические показатели — количество, глубина крепления, допустимая нагрузка, шаг, сечение. Должна быть учтена арматура – стальная одного из трех классов, композитная (стеклокомпозит, углепластик, армидокомпозит, базальтокомпозит).

Чтобы быстро провести онлайн-расчет балок перекрытий, вы можете воспользоваться специальным строительным калькулятором на нашем сервисе. В первую очередь указывают два ключевых параметра:

  1. • Длина. Показатель описывает габариты перекрываемого пролета, с небольшим запасом для монтажа на стены.
  2. • Толщина. Прочность зависит не только от стройматериала, но и от сечения.

Рекомендуемый размер сечения опоры (шарнирные, консольные, с защемлением) – от 1/25 длины и более. Общее количество можно определить с помощью нашего онлайн-калькулятора. При этом для деревянных комплектующих указываются размеры пролета и способ монтажа, определяются моменты инерции и сопротивления, модули упругости дерева и армирования, прочность на срез (двутавры, коробчатые сечения стенок).

Если есть какие-либо затруднения, наши специалисты готовы оказать всеобъемлющую консультационную поддержку.

Особенности расчета несущей способности конструкций перекрытия

В ходе подготовки проекта здания, особенно в части устройства пола и кровли, должны приниматься во внимание все факторы, сказывающиеся на нагрузке. Это требуется даже в том случае, когда используется монолитное перекрытие, наиболее прочное и долговечное. Проектные вычисления – обязательная стадия, проводимая согласно действующим стандартам и нормам. Нормативные значения различаются для квартир, лестниц, балконов, чердаков, техэтажей, террас, кровли.

Оценить несущую способность нужно в следующих ситуациях:

  1. • Увеличение веса (например, при создании надстроек).
  2. • Деформирование сооружения.
  3. • Износ стройматериалов.
  4. • Масштабная перепланировка или реконструкция.

Первые действия специалистов – анализ схемы строения, в комплексе и по отдельным частям, а также подбор крепежа. После этого оцениваются технические параметры: сечение, опоры, пролеты, степень нагрузки, величина прогиба (расчетного, относительного). По итогам всех проведенных операций подготавливается отчет.

При корректной методологии объект будут соответствовать всем нормам, повысится безопасность процесса строительства, удастся выявить все возможные риски появления дефектов. Как результат – постройка будет прочной и устойчивой, рассчитанной на десятилетия эксплуатации.

Рассчитываем вес балки, применяя онлайн калькулятор

Скачать калькулятор балки онлайн и бесплатно получить код можно на этой странице

Балка как металлический профиль

Этот вид металлопроката можно отнести к специальному, особому виду прокатных изделий, который используется для изготовления металлоконструкций, которые будут соответствовать всем необходимым техническим требованиям при эксплуатации.

Балка изготавливается из специальной стали, углеродистой или низколегированной. Способ её изготовления – при помощи литья в формы, прокат заготовки горячим или холодным способом.

Её профиль считается сложным, поэтому при её изготовлении затрачивается времени гораздо больше, чем при изготовлении, например, уголка.

Так как балка выполняет очень важную задачу, становясь основой или скелетом для будущего сооружения, к ней предъявляются особые требования, которые зависят от качества изготовления балки.

Виды балок, которые выпускаются из металлопроката

На самом деле существует несколько видов балок, но наиболее востребованными считаются: профиль в буквы «Т», называемый тавровым и профиль в виде буквы «Н» или соединёнными буквами «Т», то есть двутавровыми.

Выбирая тот или другой профиль балки, нужно просчитать наибольшую нагрузку, которую она сможет нести. Для этого используют расчеты, которые есть в формулах по сопромату. Можно использовать онлайн – калькулятор для расчета, который имеется на сайте.

В основном этот вид проката испытывает нагрузку на изгиб и нагрузку на ось. Но не нужно забывать, что при таком виде нагрузок появляется крутящийся момент, который также нужно учитывать при выборе профиля.

По данным расчета выбирают форму сечения, его размеры и материал, из которого изготовлен прокат. Площадь сечения является основным критерием расчета.

По форме сечения они бывают следующие:

  1. Обычные тавровые балки и двухскатные, которые используются между опорами, имеющими среднее расстояние друг от друга.
  2. Двутавровая балка, которая используется между опорами с максимально длинным расстоянием, имеет повышенную стойкость на изгиб.
  3. Балка с сечением в виде прямоугольника, которая используется между опорами с небольшим расстоянием друг от друга. Также применяется в случае, когда крутящийся момент на опору будет увеличен.
  4. Балка с сечением в виде буквы «Г», применяется для фасадов, применение не столь частое.

В свою очередь, двутавровые балки также имеют свои разновидности:

  • Двутавр, который имеет угол наклона граней полок 6 – 12 градусов. Изготавливается согласно ГОСТа 8239-89.
  • Двутавр с параллельными гранями полок. Он изготавливается согласно ГОСТа 26020-83 и СТО АСЧМ-20-93.
  • Двутавр специальный, который изготавливается по ГОСТу 19425-74 и делится, в свою очередь на тип «М» с углом наклоном граней до 12 градусов и тип «С» с углом наклона граней до 16 градусов.
  • Тавр изготавливается согласно ТУ 14-2-685-86.

Двутавровая балка, общий вид

Промышленность также впускает составные балки, которые изготавливаются на предприятии сварным способом или при помощи болтов.

Также этот прокат разделяется по ассортименту для удобства выбора со склада:

  • Б – стандартный вид балок;
  • Ш– широкополочный вид балок;
  • К– балки колонные двутавровые специальные.

Нормативы по которым выпускается прокат

Выпуск каждого вида проката строго регламентируется государственным стандартом, в котором указаны и размеры проката – величина углов, ширина полок, наклон граней и все размеры, которые входят в площадь поперечного сечения, а также длина проката. Кроме этого регламентируется материал, из которого он изготовлен, а также его технические характеристики.

Общие технические условия для металлопроката оговорены в ГОСТе 27772-88.

По горячекатаному двутавру из стали нужно руководствоваться ГОСТ 8239-89, который разработан для горячекатаных стальных профилей, имеющих уклон внутренних граней полок.

Сечение горячекатаной балки по ГОСТ 8239-89

Согласно ГОСТа:

  • h – высота двутавра,
  • b – ширина полки,
  • s – толщина стенки,
  • t – средняя толщина полки,
  • R – радиус внутреннего закругления,
  • r- радиус закругления полки.

В этом же документе отражены и пределы отклонений при изготовлении профиля.

На основании этого и ряда других ГОСТов был принят ГОСТ 5350-2005, который регламентирует технические условия для проката из стали углеродистой, в том числе и на прокат балки двутавровой, как стандартной, так и специального назначения. Механические свойства стали должны соответствовать таким параметрам, как временное сопротивление, предел текучести, ударной вязкости и другим параметрам, которые указаны в этом ГОСТе.

Наименование профиля двутавраВысота (h), ммШирина полки (b), ммТолщина стенки (s), ммСредняя толщина полки (t), ммМасса 1 м балки, кгМетров балки в тонне
Балка 10100554.57.29.46105.71
Балка 12120644.87.311.586.96
Балка 14140734.97.513.772.99
Балка 161608157.815.962.89
Балка 18180905.18.118.454.35
Балка 202001005.28.42147.62
Балка 222201105.48.72441.67
Балка 242401155.69.527.336.63
Балка 2727012569.831.531.75
Балка 303001356.510.236.527.4
Балка 33330140711.242.223.7
Балка 363601457.512.348.620.58
Балка 404001558.3135717.54
Балка 45450160914.266.515.04
Балка 505001701015.278.512.74
Балка 555501801116.592.610.8
Балка 606001901217.81089.26

ГОСТ 19425-74, в котором указаны параметры для выпуска специальных балок «М» и»С». Серия «М» применяется для подвесных путей, а серия «С» для оборудования шахтных проходов, причем по точности они могут изготовляться как высокой точности – маркируются буквой «А» и обычной точности- маркируются буквой «В».

Балка двутавровая по ГОСТ 19425-74. Профили и вес

Наименование профиля двутавраВысота (h), ммШирина полки (b), ммТолщина стенки (s), ммСредняя толщина полки (t), ммМасса 1 м балки, кгМетров балки в тонне
Балка 14С140805.59.116.959.17
Балка 20С200100711.427.935.84
Балка 20Са200102911.431.132.15
Балка 22С2201107.512.333.130.21
Балка 27С2701228.513.742.823.36
Балка 27Са27012410.513.74721.28
Балка 36С3601401415.871.314.03
Балка 18М1809071225.838.76
Балка 24М2401108.21438.326.11
Балка З0М30013091550.219.92
Балка 36М3601309.51657.917.27
Балка 45М45015010.51877.612.89

Балки с параллельными гранями полок имеют свой ГОСТ 26020-83

По обозначению: h – высота двутавра, b – ширина полки двутавра, s – толщина основной стенки, t – толщина полки, r – радиус сопряжения.

 

Двутавры стальные горячекатаные с параллельными гранями полок по ГОСТ 26020-83

Наименование профиля двутавраВысота (h), ммШирина полки (b), ммТолщина стенки (s), ммСредняя толщина полки (t), ммМасса 1 м балки, кгМетров балки в тонне
Нормальные двутавры
Балка 10Б1100554.18.1123.46
Балка 12Б1117.6643.88.7114.94
Балка 12Б2120644.410.496.15
Балка 14Б1137.4733.810.595.24
Балка 14Б2140734.712.977.52
Балка 16Б115782412.778.74
Балка 16Б216082515.863.29
Балка 18Б1177914.315.464.94
Балка 18Б2180915.318.853.19
Балка 20Б12001005.622.444.64
Балка 23Б12301105.625.838.76
Балка 26Б12581205.82835.71
Балка 26Б2261120631.232.05
Балка 30Б12961405.832.930.4
Балка 30Б2299140636.627.32
Балка 35Б13461556.238.925.71
Балка 35Б23491556.543.323.09
Балка 40Б1392165748.120.79
Балка 40Б23961657.554.718.28
Балка 45Б14431807.859.816.72
Балка 45Б24471808.467.514.81
Балка 50Б14922008.87313.7
Балка 50Б24962009.280.712.39
Балка 55Б15432209.58911.24
Балка 55Б25472201097.910.21
Балка 60Б159323010.5106.29.42
Балка 60Б259723011115.68.65
Балка 70Б169126012129.37.73
Балка 70Б269726012.5144.26.93
Балка 80Б179128013.5159.56.27
Балка 80Б279828014177.95.62
Балка 90Б1893300151945.15
Балка 90Б290030015.5213.84.68
Балка 100Б199032016230.64.34
Балка 100Б299832017258.23.87
Балка 100Б3100632018285.73.5
Балка 100Б4101332019.5314.53.18
Широкополочные двутавры
Балка 20Ш1193150630.632.68
Балка 23Ш12261556.536.227.62
Балка 26Ш1251180742.723.42
Балка 26Ш22551807.549.220.33
Балка 30Ш1291200853.618.66
Балка 30Ш22952008.56116.39
Балка 30Ш3299200968.314.64
Балка 35Ш13382509.575.113.32
Балка 35Ш23412501082.212.17
Балка 35Ш334525010.591.310.95
Балка 40Ш13883009.596.110.41
Балка 40Ш239230011.5111.19
Балка 40Ш339630012.5123.48.1
Балка 50Ш148430011114.48.74
Балка 50Ш248930014.5138.77.21
Балка 50Ш349530015.5156.46.39
Балка 50Ш450130016.5174.15.74
Балка 60Ш158032012142.17.04
Балка 60Ш258732016176.95.65
Балка 60Ш359632018205.54.87
Балка 60Ш460332020234.24.27
Балка 70Ш168332013.5169.95.89
Балка 70Ш269132015197.65.06
Балка 70Ш370032018235.44.25
Балка 70Ш470832020.5268.13.73
Балка 70Ш571832023305.93.27
Колонные двутавры
Балка 20К11952006.541.524.1
Балка 20К2198200746.921.32
Балка 23К1227240752.219.16
Балка 23К2230240859.516.81
Балка 26K1255260865.215.34
Балка 26K2258260973.213.66
Балка 26K32622601083.112.03
Балка 30К1296300984.811.79
Балка 30К23043001096.310.38
Балка 30К330030011.5108.99.18
Балка 35К134335010109.79.12
Балка 35К234835011125.97.94
Балка 35К335335013144.56.92
Балка 40К1393400111387.25
Балка 40К240040013165.66.04
Балка 40К340940016202.34.94
Балка 40К441940019242.24.13
Балка 40К543140023291.23.43
Двутавры дополнительной серии (Д)
Балка 24ДБ12391155.527.835.97
Балка 27ДБ1269125631.931.35
Балка 36ДБ13601457.249.120.37
Балка 35ДБ13491275.833.629.76
Балка 40ДБ13991396.239.725.19
Балка 45ДБ14501527.452.619.01
Балка 45ДБ24501807.66515.38
Балка 30ДШ1300.6201.99.472.713.76
Балка 40ДШ1397.630211.51248.06
Балка 50ДШ1496.2303.814.21556.45

Если на двутавр существуют ГОСТ ы, то изготовление тавровой балки осуществляется по ТУ 14-2-685-86

Обозначение здесь такое же, как и у двутавровой балки.

Тавры колонные и Тавры ШТ по ТУ 14-2-685-86 имеют следующие размеры

Тавры ШТ по ТУ 14-2-685-86. Наименование профиля, вес.

Наименование профиля двутавраВысота (h), ммШирина полки (b), ммТолщина стенки (s), ммСредняя толщина полки (t), ммМасса 1 м балки, кгМетров балки в тонне
Балка 13ШТ112218071021.147.39
Балка 13ШТ21241807.51224.440.98
Балка 15ШТ114220081126.637.59
Балка 15ШТ21442008.51330.233.11
Балка 15ШТ314620091533.929.5
Балка 17,5ШТ1165.52509.512.537.326.81
Балка 17,5ШТ2167250101440.824.51
Балка 17,5ШТ316925010.51645.422.03
Балка 20ШТ1190.53009.51447.820.92
Балка 20ШТ2192.530011.51655.218.12
Балка 20ШТ3194.530012.51861.316.31
Балка 25ШТ1238.5300111556.917.57
Балка 25ШТ224130014.517.568.914.51
Балка 25ШТ324430015.520.577.712.87
Балка 25ШТ424730016.523.586.611.55
Балка 30ШТ1286.5320121770.714.14
Балка 30ШТ22903201620.58012.5
Балка 30ШТ32943201824.5102.39.78
Балка 30ШТ42983202028.5116.58.58

Тавры колонные по ТУ 14-2-685-86. Название профиля и вес

Наименование профиля двутавраВысота (h), ммШирина полки (b), ммТолщина стенки (s), ммСредняя толщина полки (t), ммМасса 1 м балки, кгМетров балки в тонне
Балка 10KT1942006.51020.648.54
Балка 10KT295.5200711.523.243.1
Балка 11,5KT1110240710.525.938.61
Балка 11,5KT2111.524081229.533.9
Балка 13KT112426081232.430.86
Балка 13KT2125.5260913.536.327.55
Балка 13KT3127.52601015.541.324.21
Балка 15KT1144.5300913.542.123.75
Балка 15KT2146.53001015.547.920.88
Балка 15KT3148.53001117.554.118.48
Балка 17,5KT1168350101554.618.32
Балка 17,5KT2170.53501117.562.615.97
Балка 20KT11934001116.568.714.56
Балка 20KT2196.5400132082.412.14

Применение балок в промышленности

Балка, как наиболее мощный металлопрокат, используется в различных областях. В строительстве она выступает как основа перекрытий, перераспределяя нагрузку с перекрытия на несущие конструкции и далее на фундамент. Из неё строится основа здания, которую затем обшивают другими элементами.

Тавровая балка выдерживает меньшую нагрузку, но она также очень востребована. Балки и двойная и одинарная необходима при строительстве мостов, тоннелей, складов, ну и естественно, при строительстве зданий, как жилых, так и промышленных.

Специальные балки с повышенной прочностью используют в качестве монорельса для подъемного оборудования и для строительства туннелей шахт, при строительстве метро и тому подобных ответственных сооружений.

Одно из хороших качеств горячекатаных балок можно назвать то, что они менее подвержены коррозии по сравнению с холоднокатаными.

Балка может быть изготовлена из алюминиевого сплава, в тех случаях, когда нужна лёгкость конструкции. При этом прочность её достаточно высокая.

Поставщики металлопроката

В России есть много металлургических заводов, около 60, но балки выпускают только некоторые из них.

Например, выпускает балку Алапаевский металлургический завод в числе остальных прокатных изделий, Магнитогорский металлургический комбинат выпускает балку горячекатаную, Белорецкий металлургический комбинат, Челябинский металлургический комбинат, Петровск – Забайкальский завод, Оскольский электрометаллургический завод, Омутнинский металлургический комбинат. Другие предприятия выпускают этот прокат при наличии соответствующего заказа, конечно заказ должен быть большим. Так как балка это продукция со специфическими свойствами, её иногда закупают за рубежом.

Заказ балки можно сделать как на предприятии, так и у металлотрейдеров, поставляющих металлопрокат, в Москве их есть много. Желательно работать с крупными организациями, у которых высокий рейтинг.

Для заказа продукции нужно высчитать вес балки. По приведенным здесь размерам выбираете вес одного погонного метра балки нужно вам профиля. Потом вес 1 метра погонного умножаем на длину проката, то есть балки. Для простоты расчета предлагаем использовать наш онлайн калькулятор веса, пользоваться которым очень просто и надёжно. Результат получаете мгновенно.

ENGINEERING.com | Калькуляторы прогиба балки

Калькуляторы прогиба балки — сплошные прямоугольные балки, полые прямоугольные балки, сплошные круглые балки

Введите значение и нажмите «Рассчитать». Результат будет отображаться

Расчет прогиба для сплошных прямоугольных балок
Расчет прогиба для полых прямоугольных балок
Расчет прогиба для сплошных круглых балок
Расчет прогиба для круглых трубчатых балок

Расчет прогиба сплошных прямоугольных балок

фунтов стерлингов

Введите свои значения:
Длина:

Дюймы

Ширина:

Дюймы

Высота:

Дюймы

Сила:
Материал:
Результатов:
Прогиб:

Дюймы

Напряжение изгиба:

PSI


Расчет прогиба полых прямоугольных балок

фунтов стерлингов

Введите свои значения:
Длина:

Дюймы

Ширина:

Дюймы

Высота:

Дюймы

Толщина стенки:

Дюймы

Сила:
Материал:
Результат:
Прогиб:

Дюймы

Напряжение изгиба:

PSI


Расчет прогиба сплошных круглых балок

фунтов стерлингов

Введите свои значения:
Длина:

Дюймы

Диаметр:

Дюймы

Сила:
Материал:
Результатов:
Прогиб:

Дюймы

Напряжение изгиба:

PSI


Расчет прогиба для круглых трубных балок

фунтов стерлингов

Введите свои значения:
Длина:

Дюймы

Диаметр:

Дюймы

Толщина стенки:

Дюймы

Сила:
Материал:
Результат:
Прогиб:

Дюймы

Напряжение изгиба:

PSI

StructX — Формулы расчета балок

Формулы расчета балок

Просто выберите изображение, которое больше всего соответствует конфигурации балки и условиям нагрузки, которые вас интересуют, чтобы получить подробный обзор всех структурных свойств.Уравнения балки для результирующих сил, поперечных сил, изгибающих моментов и прогиба можно найти для каждого показанного случая балки. Для проектирования и оценки балок в метрических и дюймовых единицах предусмотрены удобные калькуляторы.

Сборник электронных таблиц по проектированию конструкций для расчета балок с использованием Excel доступен для покупки и находится под каждым типом балок.

Дополнительную информацию о теории проектирования балок и сделанных допущениях можно найти здесь.

Простая балка с UDL

Простая балка с UIL

Простая балка с центральной УИЛ

Простая балка с PDUL

Простая балка с PDUL на одном конце

Простая балка с PDUL на каждом конце

Простая балка с PL в центре

Простая балка с PL в любой точке

Простая балка с одинаково расположенными PL

Балка с неравномерно разнесенными пластинами

Балка с неравномерно разнесенными ВИП

Простая балка с UDL и EM

Простая балка с PL и EM

Фиксированная торцевая балка с UDL

Фиксированная торцевая балка с центральным PL

Фиксированная торцевая балка.PL в любой точке

Фиксированная балка с UDL

Фиксированная балка с центральным PL

Фиксированная балка с PL в любой точке

Консольная балка с UIL

Консольная балка с UDL

Консольная балка с UDL и EM

Консольная балка. PL в любой точке

Консольная балка с PL на свободном конце

Консольная балка с PL и EM

Балка навесная с UDL

Свесная балка с UDL на конце

Балка навесная с PL на конце

Балка навесная с деталью UDL

Свисающая балка PL в любой точке

Балка с двумя свесами и UDL

Двухпролетная балка с частичным UDL

Двухпролетная балка с PL

Двухпролетная балка.PL в любой точке

Двухпролетная балка с UDL

Двухпролетная балка с двумя PL

Два неравных пролета с UDL

Два неравных пролета с PL

Трехпролетная балка с частичным UDL

Трехпролетная балка с концевыми UDL

Трехпролетная балка с UDL

Четырехпролетная балка. Незагруженный пролет

Четырехпролетная балка.Разгрузочные пролеты

Четырехпролетная балка с UDL

Калькулятор стальной балки

| Калькулятор стальных балок для проектирования стальных балок

1.2 Выбрать стальную балку

Рассчитайте стальные балки, способные выдерживать указанные ниже нагрузки (расчеты по расчету допустимого напряжения AISC, 9-е издание)

W балки

S балки

Американские стандартные каналы

Выберите конкретную стальную балку

W4X13W5X16W5X19W6X9W6X12W6X20W6X16W6X25W8X10W8X24W8X13W8X31W8X28W8X15W8X35W8X18W8X40W8X21W8X48W8X58W8X67W10X33W10X12W10X39W10X15W10X49W10X45W10X54W10X17W10X19W10X22W10X60W10X26W10X68W10X30W10X77W10X88W10X100W10X112W12X40W12X14W12X16W12X65W12X53W12X45W12X50W12X58W12X19W12X26W12X22W12X30W12X72W12X79W12X87W12X35W12X96W12X106W12X120W12X136W14X22W14X43W14X48W14X30W14X26W14X61W14X53W14X68W14X34W14X90W14X38W14X74W14X99W14X109W14X82W14X120W14X132W16X26W16X36W16X31W16X40W16X45W16X50W16X67W16X57W16X77W16X89W16X100W18X35W18X40W18X50W18X46W18X55W18X60W18X76W18X86W18X65W18X71W18X97W18X106W18X119W18X130W18X143W18X158W18X175W18X192W18X211W21X44W21X50W21X62W21X68W21X57W18X234W21X73W21X83W21X101W21X111W18X258W21X93W21X122W21X132W18X283W21X147W18X311W21X166W21X182W21X201W24X55W24X68W24X62W24X76W24X84W24X104W24X117W24X94W24X103W24X131W24X146W24X162W24X176W24X192W24X207W24X229W24X250W24X279W27X84W27X94W24X306W27X102W27X114W27X146W24X335W27X129W27X161W27X178W24X370W27X194W27X217W27X235W27X258W27X281W30X90W 27X307W30X99W30X108W30X116W27X336W30X124W30X132W30X173W27X368W30X148W30X191W30X211W30X235W30X261W30X292W30X326W27X539W30X357W33X118W33X130W30X391W33X141W33X152W33X201W33X169W33X221W33X241W33X263W33X291W33X318W33X354W36X135W36X150W33X387W36X160W36X170W36X182W36X194W36X210W36X232W36X256W40X149W40X167W40X199W40X183W40X215W40X249W40X277W40X297W40X324W40X362W40X397W40X593S3X7.5S3X5.7S4X9.5S4X7.7S5X10S6X17.25S6X12.5S8X23S8X18.4S10X35S10X25.4S12X50S12X40.8S12X35S12X31.8S15X50S15X42.9S18X70S18X54.7S20X75S20X66S20X96S20X86S24X100S24X90S24X80S24X121S24X106C3X6C3X5C3X4.1C4X7.25C4X5.4C5X9C5X6.7C6X13C6X10.5C6X8.2C7X14.75C7X12.25C7X9.8C8X18.75C8X13.75C8X11.5C9X20C9X15C9X13.4C10X30C10X25C10X20C10X15. 3C12X30C12X25C12X20.7C15X50C15X40C15X33.9 Глубина (дюйм) x вес (фунт / фут)

Beam Калькулятор — Инструменты для инженера

Сталь раздел — CISC

W1100X499W1100X433W1100X390W1100X343W1000X883W1000X748W1000X642W1000X591W1000X554W1000X539W1000X483W1000X443W1000X412W1000X371W1000X321W1000X296W1000X584W1000X494W1000X486W1000X438W1000X415W1000X393W1000X350W1000X314W1000X272W1000X249W1000X222W920X1191W920X970W920X787W920X725W920X656W920X588W920X537W920X491W920X449W920X420W920X390W920X368W920X344W920X381W920X345W920X313W920X289W920X271W920X253W920X238W920X223W920X201W840X576W840X527W840X473W840X433W840X392W840X359W840X329W840X299W840X251W840X226W840X210W840X193W840X176W760X582W760X531W760X484W760X434W760X389W760X350W760X314W760X284W760X257W760X220W760X196W760X185W760X173W760X161W760X147W760X134W690X802W690X548W690X500W690X457W690X419W690X384W690X350W690X323W690X289W690X265W690X240W690X217W690X192W690X170W690X152W690X140W690X125W610X551W610X498W610X455W610X415W610X372W610X341W610X307W610X285W610X262W610X241W610X217W610X195W610X174W610X155W610X15 3W610X140W610X125W610X113W610X101W610X91W610X84W610X92W610X82W530X300W530X272W530X248W530X219W530X196W530X182W530X165W530X150W530X138W530X123W530X109W530X101W530X92W530X82W530X72W530X85W530X74W530X66W460X464W460X421W460X384W460X349W460X315W460X286W460X260W460X235W460X213W460X193W460X177W460X158W460X144W460X128W460X113W460X106W460X97W460X89W460X82W460X74W460X67W460X61W460X68W460X60W460X52W410X149W410X132W410X114W410X100W410X85W410X74W410X67W410X60W410X54W410X46W410X39W360X1086W360X990W360X900W360X818W360X744W360X677W360X634W360X592W360X551W360X509W360X463W360X421W360X382W360X347W360X314W360X287W360X262W360X237W360X216W360X196W360X179W360X162W360X147W360X134W360X122W360X110W360X101W360X91W360X79W360X72W360X64W360X57W360X51W360X45W360X39W360X33W310X500W310X454W310X415W310X375W310X342W310X313W310X283W310X253W310X226W310X202W310X179W310X158W310X143W310X129W310X118W310X107W310X97W310X86W310X79W310X74W310X67W310X60W310X52W310X45W310X39W310X31W310X33W310X28W310X24W310X21W250X167W250X149W250X13 1W250X115W250X101W250X89W250X80W250X73W250X67W250X58W250X49W250X45W250X39W250X33W250X24W250X28W250X25W250X22W250X18W200X100W200X86W200X71W200X59W200X52W200X46W200X42W200X36W200X31W200X27W200X21W200X22W200X19W200X15W150X37W150X30W150X22W150X24W150X18W150X14W150X13W130X28W130X24W100X19S610X180S610X158S610X149S610X134S610X119S510X143S510X128S510X112S510X98-2S460X104S460X81-4S380X74S380X64S310X74S310X60-7S310X52S310X47S250X52S250X38S200X34S200X27S150X26S150X19S130X15S100X14-1S100X11S75X11S75X8M310X17-6M310X16-1M310X14-9M250X13-4M250X11-9M250X11-2M200X9-7M200X9-2M150X6-6M150X5-5M130X28-1M100X8-9HP360X174HP360X152HP360X132HP360X108HP310X125HP310X110HP310X94HP310X79HP250X85HP250X62HP200X54WWF2000X732WWF2000X648WWF2000X607WWF2000X542WWF1800X700WWF1800X659WWF1800X617WWF1800X575WWF1800X510WWF1600X622WWF1600X580WWF1600X538WWF1600X496WWF1600X431WWF1400X597WWF1400X513WWF1400X471WWF1400X405WWF1400X358WWF1200X487WWF1200X418WWF1200X380WWF1200X333WWF1200X302WWF1200X263WWF1100X458WWF1100X388WWF1100X351W WF1100X304WWF1100X273WWF1100X234WWF1000X447WWF1000X377WWF1000X340WWF1000X293WWF1000X262WWF1000X223WWF1000X200WWF900X417WWF900X347WWF900X309WWF900X262WWF900X231WWF900X192WWF900X169WWF800X339WWF800X300WWF800X253WWF800X223WWF800X184WWF800X161WWF700X245WWF700X214WWF700X196WWF700X175WWF700X152WWF650X864WWF650X739WWF650X598WWF650X499WWF650X400WWF600X793WWF600X680WWF600X551WWF600X460WWF600X369WWF550X721WWF550X620WWF550X503WWF550X420WWF550X280WWF500X651WWF500X561WWF500X456WWF500X381WWF500X343WWF500X306WWF500X276WWF500X254WWF500X223WWF500X197WWF450X503WWF450X409WWF450X342WWF450X308WWF450X274WWF450X248WWF450X228WWF450X201WWF450X177WWF400X444WWF400X362WWF400X303WWF400X273WWF400X243WWF400X220WWF400X202WWF400X178WWF400X157WWF350X315WWF350X263WWF350X238WWF350X212WWF350X192WWF350X176WWF350X155WWF350X137C380X74C380X60C380X50C310X45C310X37C310X31C250X45C250X37C250X30C250X23C230X30C230X22C230X20C200X28C200X21C200X17C180X22C180X18C180X15C150X19C150X16C150X12C130X13C130X10C100X11C100X9C100X8C100X7C75X9C75 X7C75X6C75X5MC460X86MC460X77-2MC460X68-2MC460X63-5MC330X74MC330X60MC330X52MC330X47-3MC310X74MC310X67MC310X60MC310X52MC310X46MC310X15-8MC250X61-2MC250X50MC250X42-4MC250X37MC250X33MC250X12-5MC230X37-8MC230X35-6MC200X33-9MC200X31-8MC200X29-8MC200X27-8MC200X12-6MC180X33-8MC180X28-4MC150X26-8MC150X22-8MC150X24-3MC150X22-5MC150X17-9L203X203X29L203X203X25L203X203X22L203X203X19L203X203X16L203X203X14L203X203X13L203X152X25L203X152X22L203X152X19L203X152X16L203X152X14L203X152X13L203X102X25L203X102X19L203X102X13L178X102X19L178X102X16L178X102X13L178X102X11L178X102X9-5L152X152X25L152X152X22L152X152X19L152X152X16L152X152X14L152X152X13L152X152X11L152X152X9-5L152X152X7- 9L152X152X6-4L152X102X22L152X102X19L152X102X16L152X102X14L152X102X13L152X102X11L152X102X9-5L152X102X7-9L152X89X16L152X89X13L152X89X9-5L152X89X7-9L127X127X22L127X127X19L127X127X16L127X127X13L127X127X11L127X127X9-5L127X127X7-9L127X127X6-4L127X89X19L127X89X16L127X89X13L127X89X9-5L127X89X7-9L127X89X6-4L127X76X13L127X76X11L127X76X9-5L127X76X7- 9L127X76X6-4L102X102X19L102X102X16L102X102X13L102X102X11L102X102X9-5L102X102X7-9L102X102X6-4L102X89X13L102X89X11L102X89X9-5L102X89X7-9L102X89X6-4L102X76X16L102X76X13L102X76X11L102X76X9-5L102X76X7-9L102X76X6-4L89X89X13L89X89X11L89X89X9-5L89X89X7-9L89X89X6-4L89X76X13L89X76X9-5L89X76X7-9L89X76X6-4L89X64X13L89X64X9-5L89X64X7-9L89X64X6-4L76X76X13L76X76X11L76X76X9-5L76X76X7-9L76X76X6-4L76X76X4-8L76X64X13L76X64X9-5L76X64X7- 9L76X64X6-4L76X64X4-8L76X51X13L76X51X9-5L76X51X7-9L76X51X6-4L76X51X4-8L64X64X13L64X64X9-5L64X64X7-9L64X64X6-4L64X64X4-8L64X51X9-5L64X51X7-9L64X51X6-4L64X51X4-8L51X51X9-5L51X51X7-9L51X51X6-4L51X51X4-8L51X51X3-2L51X38X6-4L51X38X4-8L51X38X3-2L44X44X6-4L44X44X4-8L44X44X3- 2L38X38X6-4L38X38X4-8L38X38X3-2L32X32X6-4L32X32X4-8L32X32X3-2L25X25X6-4L25X25X4-8L25X25X3-2L19X19X3-2WT460X223WT460X208-5WT460X193-5WT460X182-5WT460X171WT460X156-5WT460X144-5WT460X135-5WT460X126-5WT460X119WT460X111-5WT460X100-5WT420X179-5WT420X164-5WT420X149-5WT420X113WT420X105WT420X96-5WT420X88WT380X157WT380X1 42WT380X128-5WT380X98WT380X92-5WT380X86-5WT380X80-5WT380X73-5WT345X132-5WT345X120WT345X108-5WT345X85WT345X76WT345X70WT345X62-5WT305X120-5WT305X108-5WT305X97-5WT305X87WT305X77-5WT305X70WT305X62-5WT305X56-5WT305X50-5WT305X46WT305X41WT265X109-5WT265X98WT265X91WT265X82-5WT265X75WT265X69WT265X61-5WT265X54-5WT265X50-5WT265X46WT265X41WT265X42-5WT265X37WT265X33WT230X88-5WT230X79WT230X72WT230X64WT230X56-5WT230X53WT230X48-5WT230X44- 5WT230X41WT230X37WT230X34WT230X30WT230X26WT205X74-5WT205X66WT205X57WT205X50WT205X42-5WT205X37WT205X33-5WT205X30WT205X27WT205X23WT205X19-5WT180X543WT180X495WT180X450WT180X409WT180X372WT180X338-5WT180X317WT180X296WT180X275-5WT180X254-5WT180X231-5WT180X210-5WT180X191WT180X173-5WT180X157WT180X143-5WT180X131WT180X118-5WT180X108WT180X98WT180X89-5WT180X81WT180X73-5WT180X67WT180X61WT180X55WT180X50-5WT180X45-5WT180X39-5WT180X36WT180X32WT180X28-5WT180X25-5WT180X22-5WT180X19-5WT180X16-5WT155X250WT155X227WT155X207-5WT155X187-5WT155X171WT155X156- 5WT155X141-5WT155X126-5WT155X113WT1 55X101WT155X89-5WT155X79WT155X71-5WT155X64-5WT155X59WT155X53-5WT155X48-5WT155X43WT155X39-5WT155X37WT155X33-5WT155X30WT155X26WT155X22-5WT155X19-5WT155X16-5WT155X14WT155X12WT155X10-5WT125X83-5WT125X74-5WT125X65-5WT125X57-5WT125X50-5WT125X44-5WT125X40WT125X36-5WT125X33-5WT125X29WT125X24-5WT125X22-5WT125X19-5WT125X16-5WT125X14WT125X12-5WT125X11WT125X9WT100X50WT100X43WT100X35- 5WT100X29-5WT100X26WT100X23WT100X21WT100X18WT100X15-5WT100X13-5WT100X11WT100X9-5WT100X7-5WT75X18-5WT75X15WT75X11WT75X12WT75X9WT75X7WT65X14WT65X12WT50X9-5WWT275X360-5WWT275X310WWT275X251-5WWT275X210WWT275X140WWT250X325-5WWT250X280-5WWT250X228WWT250X190-5WWT250X171-5WWT250X153WWT250X138WWT250X127WWT250X111-5WWT250X98-5WWT225X251-5WWT225X204-5WWT225X171WWT225X154WWT225X137WWT225X124WWT225X114WWT225X100-5WWT225X88-5WWT200X222WWT200X181WWT200X151-5WWT200X136-5WWT200X121-5WWT200X110WWT200X101WWT200X89WWT200X78-5WWT175X157-5WWT175X131- 5WWT175X119WWT175X106WWT175X96WWT175X88WWT175X77-5WWT175X68-52L203X152X252L203X152X222L203X1 52X192L203X152X162L203X152X142L203X152X132L203X102X252L203X102X192L203X102X132L178X102X192L178X102X162L178X102X132L178X102X112L178X102X9-52L152X102X222L152X102X192L152X102X162L152X102X142L152X102X132L152X102X112L152X102X9-52L152X102X7-92L152X89X162L152X89X132L152X89X9-52L152X89X7-92L127X89X192L127X89X162L127X89X132L127X89X9-52L127X89X7-92L127X89X6-42L127X76X132L127X76X112L127X76X9-52L127X76X7-92L127X76X6-42L102X89X132L102X89X112L102X89X9-52L102X89X7-92L102X89X6-42L102X76X162L102X76X132L102X76X112L102X76X9-52L102X76X7-92L102X76X6-42L89X76X132L89X76X9-52L89X76X7-92L89X76X6-42L89X64X132L89X64X9-52L89X64X7-92L89X64X6-42L76X64X132L76X64X9-52L76X64X7- 92L76X64X6-42L76X64X4-82L76X51X132L76X51X9-52L203X203X292L203X203X252L203X203X222L203X203X192L203X203X162L203X203X142L203X203X132L76X51X7-92L76X51X6-42L76X51X4-82L64X51X9-52L64X51X7-92L64X51X6-42L64X51X4-82L51X38X6-42L51X38X4-82L51X38X3-22L152X152X252L152X152X222L152X152X192L152X152X162L152X152X142L152X152X132L152X152X112L152X152X9-52L152X152X7- 92L152X152X6-42L127X127X222L127X127X192L127X127X162L127X127X132L127X127X112L127X127X9-52L127X127X7-92L127X127X6-42L102X102X192L102X102X162L102X102X132L102X102X112L102X102X9-52L102X102X7-92L102X102X6-42L89X89X132L89X89X112L89X89X9-52L89X89X7-92L89X89X6-42L76X76X132L76X76X112L76X76X9-52L76X76X7-92L76X76X6-42L76X76X4-82L64X64X132L64X64X9-52L64X64X7-92L64X64X6-42L64X64X4-82L51X51X9-52L51X51X7-92L51X51X6-42L51X51X4-82L51X51X3-22L44X44X6-42L44X44X4- 82L44X44X3-22L38X38X6-42L38X38X4-82L38X38X3-22L32X32X6-42L32X32X4-82L32X32X3-22L25X25X6-42L25X25X4-82L25X25X3-22L19X19X3-2HS305X305X16HS305X305X13HS305X305X9-5HS305X305X8-0HS305X305X6-4HS254X254X16HS254X254X13HS254X254X9-5HS254X254X8-0HS254X254X6-4HS203X203X16HS203X203X13HS203X203X9-5HS203X203X8-0HS203X203X6-4HS178X178X16HS178X178X13HS178X178X9-5HS178X178X8-0HS178X178X6-4HS178X178X4-8HS152X152X13HS152X152X9- 5HS152X152X8-0HS152X152X6-4HS152X152X4-8HS127X127X13HS127X127X9-5HS127X127X8-0HS127X127X6-4HS127X127X4-8HS114X114X13HS114X114X9-5HS8-0114X114X HS114X114X6-4HS114X114X4-8HS114X114X3-2HS102X102X13HS102X102X9-5HS102X102X8-0HS102X102X6-4HS102X102X4-8HS102X102X3-2HS89X89X9-5HS89X89X8-0HS89X89X6-4HS89X89X4-8HS89X89X3-2HS76X76X9-5HS76X76X8-0HS76X76X6-4HS76X76X4-8HS76X76X3-2HS64X64X8-0HS64X64X6-4HS64X64X4-8HS64X64X3-2HS51X51X6-4HS51X51X4-8HS51X51X3- 2HS38X38X4-8HS38X38X3-2HS356X254X16HS356X254X13HS356X254X9-5HS305X203X16HS305X203X13HS305X203X9-5HS305X203X8-0HS305X203X6-4HS254X152X16HS254X152X13HS254X152X9-5HS254X152X8-0HS254X152X6-4HS203X152X13HS203X152X9-5HS203X152X8-0HS203X152X6-4HS203X152X4-8HS203X102X13HS203X102X9-5HS203X102X8-0HS203X102X6-4HS203X102X4-8HS178X127X13HS178X127X9-5HS178X127X8-0HS178X127X6-4HS178X127X4-8HS152X102X13HS152X102X9-5HS152X102X8-0HS152X102X6-4HS152X102X4- 8HS152X76X13HS152X76X9-5HS152X76X8-0HS152X76X6-4HS152X76X4-8HS127X76X9-5HS127X76X8-0HS127X76X6-4HS127X76X4-8HS102X76X9-5HS102X76X8-0HS102X76X6-4HS102X76X4-8HS102X76X3-2HS102X51X9-5HS102X51X8-0HS102X51X6-4HS102X51X4-8HS102X51X3-2HS89X64X8-0HS89X64X6-4HS89X64 X4-8HS89X64X3-2HS76X51X8-0HS76X51X6-4HS76X51X4-8HS76X51X3-2HS51X25X4-8HS51X25X3-2HS406X13HS406X9-5HS406X6-4HS356X16HS356X13HS356X9-5HS356X6-4HS324X13HS324X9-5HS324X6-4HS273X13HS273X6-4HS273X4-8HS219X16HS219X13HS219X9-5HS219X6-4HS219X4-8HS178X13HS178X9-5HS178X8-0HS178X6-4HS178X4-8HS168X13HS168X9-5HS168X8- 0HS168X6-4HS168X4-8HS168X3-2HS152X9-5HS152X8-0HS152X6-4HS152X4-8HS152X3-2HS141X9-5HS141X6-4HS141X4-8HS127X13HS127X9-5HS127X8-0HS127X6-4HS127X4-8HS127X3-2HS114X9-5HS114X4-8HS114X3-2HS102X8-0HS102X6-4HS102X4-8HS102X3-2HS89X8-0HS89X6- 4HS89X4-8HS89X3-2HS76X6-4HS76X4-8HS73X6-4HS73X4-8HS73X3-2HS64X6-4HS64X4-8HS64X3-2HS60X6-4HS60X4-8HS60X3-2HS48X4-8HS48X3-2

Онлайн-калькуляторы для проектирования конструкций


Загрузка

Анализ снеговой нагрузки
Кодекс ASCE 7-05 для зданий с плоской или пологой крышей — для сбалансированной снеговой нагрузки, сноса и дополнительных нагрузок от дождя на снегу

Анализ ледовой нагрузки (формы WT, MT и ST)
Кодекс ASCE 7-05 — Глава 10- для ледовых нагрузок из-за ледяного дождя на формах WT, MT и ST

Анализ ледовой нагрузки (формы W, M, S и HP)
Код ASCE 7-05 — Глава 10 — для ледовых нагрузок из-за ледяного дождя на формах W, M, S и HP

Анализ ледовой нагрузки (формы C и MC)
Код ASCE 7-05 — Глава 10 для ледовых нагрузок из-за ледяного дождя на формах C и MC

Сейсмический сдвиг основания (одноуровневые здания)
Спецификации IBC2006 и ASCE 7-05 — Процедура использования эквивалентной боковой силы для обычных одноуровневых систем зданий / конструкций

Анализ ветровой нагрузки (малоэтажные здания)
Кодекс ASCE 7-05 для закрытых или частично закрытых зданий с использованием метода 2: аналитическая процедура (раздел 6.5) для малоэтажной застройки


Анализ нагрузки

Тепловые эффекты для стальных зданий
Для балок с опорой на грунт, комбинированных опор, полос перекрытий или полос матов предполагаемой конечной длины с обоими свободными концами


Фонды

Расчет балки на упругом основании (BOEF)
Для балок с опорой на грунт, комбинированных опор, полосы перекрытия или полосы мата предполагаемой конечной длины со свободными обоими концами

Бетонная плита на основе анализа толщины
для плиты, подвергшейся концентрированной последующей нагрузке (для k = 100 pci)
в соответствии с PCA «Расчет толщины плиты для промышленных бетонных полов на уровне уклона»

Бетонная плита уровня
для плиты, подверженной внутренней концентрированной стойке или колесной нагрузке
Предполагается, что плита армирована только с учетом усадки и температуры

Бетонная плита по анализу уклона
Для плиты, подвергающейся непрерывной линейной нагрузке от стены


Элемент дизайна

Допустимая осевая нагрузка для отдельных пластин
на основе полного сечения, нагруженного равномерно при растяжении или сжатии
Сжатие в соответствии с руководством AISC 9-го издания (ASD)

Анализ ребра жесткости стенки стальной балки
Критерии упругости, деформации, продольного изгиба и ребра жесткости для сосредоточенной нагрузки или реакции
Согласно AISC 9-е издание Руководства (ASD)

Анализ стальных балок и колонн / проверка кода
Проверка кода напряжения в соответствии с AISC 9-е издание руководства (ASD)
для форм W, S, M и HP

Анализ стальных балок
Общий стандартный анализ балок для стальных балок, рассматриваемых как однопролетные балки
, подверженные нестандартным нагрузкам


Конструкция рамы

Расчет на изгиб X-образных поперечных связей для боковых нагрузок (1-этажные здания)
Для 1-этажного изгиба — при условии, что система полностью скреплена, только натяжение

Анализ изгиба X-образных скоб для боковых нагрузок (2-этажные здания)
для 2-этажных Согнутый — при условии, что система полностью скреплена, только натяжение



Заявление об отказе от ответственности: Этот калькулятор не предназначен для использования для проектирования реальных конструкций, а только для схематического (предварительного) понимания принципов структурного проектирования.Для проектирования реальной конструкции следует проконсультироваться с компетентным специалистом.

«Расчеты любезно предоставлены Алексом Томановичем, ЧП»

Онлайн-конструкторское проектирование

Бесплатно

Расчет закрепленной балки (дюймовая)
Расчет бесплатный, логин не требуется

Расчет внутренних сил балки (поперечная сила, изгибающий момент) и прогибов

имперский

луч

приколот

грузы

случаи нагрузки

силы

отклонение

Открыть расчетный лист

Бесплатно

Балка, фиксированная на обоих концах (дюймовые)
Расчет бесплатный, логин не требуется

Расчет внутренних сил балки (поперечная сила, изгибающий момент) и прогибов

имперский

луч

фиксированный

грузы

случаи нагрузки

силы

отклонение

Открыть расчетный лист

Бесплатно

Расчет закрепленной балки (метрическая система)
Расчет бесплатный, логин не требуется

Расчет внутренних сил балки (поперечная сила, изгибающий момент) и прогибов

метрика

луч

грузы

случаи нагрузки

силы

отклонение

Открыть расчетный лист

Бесплатно

Балка, закрепленная на обоих концах (метрическая система)
Расчет бесплатный, логин не требуется

Расчет внутренних сил балки (поперечная сила, изгибающий момент) и прогибов

метрика

луч

фиксированный

грузы

случаи нагрузки

силы

отклонение

Открыть расчетный лист

Бесплатно

Емкость балки RC (EC2)
Бесплатно, на ограниченный период, требуется логин

Расчет прочности на изгибающий момент железобетонной балки (Еврокод 2)

метрика

EC2

луч

конкретный

Открыть расчетный лист
Предварительный просмотр

Бесплатно

Допустимая нагрузка на изгиб стальной балки (дюймовая)
Бесплатно, на ограниченный период, требуется логин

Расчет прочности при изгибе стальной балки и поперечной устойчивости при кручении (AISC, LRFD)

имперский

луч

изгиб

стали

LRFD

AISC

Открыть расчетный лист
Предварительный просмотр

Бесплатно

Стальной элемент жесткости подшипника балки (дюймовая)
Бесплатно, на ограниченный период

Проверьте требования к опорному элементу жесткости для стенок с сосредоточенными силами; Веб-локальная урожайность; Web Crippling; Боковое изгибание полотна

имперский

луч

сеть

уступающий

калечащий

коробление

LRFD

AISC

Открыть расчетный лист
Предварительный просмотр

Бесплатно

Диаметр балки (EC5)
Бесплатно, на ограниченный период, требуется логин

Расчет несущей способности деревянных балок, проверка деревянных элементов (Еврокод 5)

метрика

EC5

луч

древесина

изгиб

Открыть расчетный лист
Предварительный просмотр

Бесплатно

Максимальный диаметр балки RC (ACI318)
Бесплатно, на ограниченный период, требуется логин

Расчет прочности на изгибающий момент железобетонной балки (ACI 318)

имперский

ACI318

луч

изгиб

конкретный

Открыть расчетный лист
Предварительный просмотр

Бесплатно

Простая балка — равномерно распределенная нагрузка
Расчет бесплатный, логин не требуется

Расчет сдвигов, моментов и прогибов для простой опорной балки при равномерно распределенной нагрузке

метрика

статика

грузы

силы

луч

Открыть расчетный лист

Бесплатно

Простая балка — сосредоточенная нагрузка в центре
Расчет бесплатный, логин не требуется

Расчет сдвигов, моментов и прогибов для простой опорной балки с сосредоточенной нагрузкой в ​​центре

метрика

статика

грузы

силы

луч

Открыть расчетный лист

Бесплатно

Простая балка — сосредоточенная нагрузка в любой точке
Расчет бесплатный, логин не требуется

Расчет сдвигов, моментов и прогибов для простой опорной балки, сосредоточенной нагрузки в любой точке

метрика

статика

грузы

силы

луч

Открыть расчетный лист

Бесплатно

Простая балка 2 Концентрированная сим.грузы
Расчет бесплатный, логин не требуется

Расчет сдвигов, моментов и прогибов для простой опорной балки, 2 сосредоточенных симметричных нагрузки

метрика

статика

грузы

силы

луч

Открыть расчетный лист

Бесплатно

Простая балка 2 Концентрированная сим.грузы
Расчет бесплатный, логин не требуется

Расчет сдвигов, моментов и прогибов для простой опорной балки, 2 сосредоточенных симметричных нагрузки

имперский

статика

грузы

силы

луч

Открыть расчетный лист

Бесплатно

Простая балка — равномерно распределенная нагрузка
Расчет бесплатный, логин не требуется

Расчет сдвигов, моментов и прогибов для простой опорной балки при равномерно распределенной нагрузке

имперский

статика

грузы

силы

луч

Открыть расчетный лист

Расчет балки Часть I | Онлайн-калькулятор

В этом разделе вы можете выполнить онлайн-расчет балок при сосредоточенной нагрузке.Расчеты определяют прогиб, угол поворота и изгибающий момент в произвольной заданной точке балки при различных граничных условиях.

Исходные данные:

L — длина балки, миллиметр;

а — координата точки приложения сосредоточенной нагрузки, миллиметры;

X — координата точки решения, миллиметры;

F — нагрузка, ньютоны;

I x — момент инерции секции, м 4 ;

Е — модуль упругости материала балки, паскаль

Расчет балки №1.1

Расчет консольной балки при сосредоточенной нагрузке.

Граничные условия:

R L = 0 — реакция опоры в крайней левой точке;

M L = 0 — изгибающий момент в крайней левой точке;

θ R = 0 — угол поворота в крайней правой точке;

Y R = 0 — прогиб балки в крайней правой точке.

Расчет балки # 2.1

Расчет балки с зажатым концом и скользящей опорой при сосредоточенной нагрузке.

Граничные условия:

R L = 0 — реакция опоры в крайней левой точке;

θ L = 0 — угол поворота в крайней левой точке;

θ R = 0 — угол поворота в крайней правой точке;

Y R = 0 — прогиб в крайней правой точке.

Расчет балки № 3.1

Расчет балки с зажатым концом и шарнирной опорой при сосредоточенной нагрузке.

Граничные условия:

М L = 0 — изгибающий момент в крайней левой точке;

Y L = 0 — прогиб в крайней левой точке;

θ R = 0 — угол поворота в крайней правой точке;

Y R = 0 — прогиб в крайней правой точке.

Расчет балки № 4.1

Расчет балки с защемленными концами при сосредоточенной нагрузке.

Граничные условия:

θ L = 0 — угол поворота в крайней левой точке;

Y L = 0 — прогиб в крайней левой точке;

θ R = 0 — угол поворота в крайней правой точке;

Y R = 0 — прогиб в крайней правой точке.

Бесплатный калькулятор луча | Калькулятор изгибающего момента, поперечной силы и прогиба

Добро пожаловать в наш бесплатный онлайн-калькулятор диаграмм изгибающего момента и поперечной силы, который может генерировать диаграммы реакций, поперечных сил (SFD) и изгибающих моментов (BMD) консольной балки или просто поддерживаемой балки. Используйте этот калькулятор пролета балки, чтобы определить реакции на опоры, построить диаграмму сдвига и момента для балки и рассчитать прогиб стальной или деревянной балки. Бесплатный онлайн-калькулятор балки для создания реакций, расчета прогиба стальной или деревянной балки, построения диаграмм сдвига и момента балки.Это бесплатная версия нашего полного программного обеспечения SkyCiv Beam. Доступ к нему можно получить из любой из наших Платных учетных записей, которая также включает в себя полное программное обеспечение для структурного анализа.

Используйте интерактивное окно выше, чтобы просмотреть и удалить длину балки, опоры и добавленные нагрузки. Любые внесенные изменения автоматически перерисовывают диаграмму свободного тела для любой балки с опорой или консольной балкой. Калькулятор реакции балки и расчет изгибающего момента запускаются после нажатия кнопки «Решить» и автоматически генерируют диаграммы сдвига и изгибающего момента.Вы также можете щелкнуть отдельные элементы этого калькулятора балки LVL, чтобы редактировать модель.

Калькулятор пролета балки легко рассчитает реакции на опорах. Он может рассчитывать реакции на опорах консольных или простых балок. Это включает в себя расчет реакций консольной балки, которая имеет изгибающий момент, а также силы реакции x, y.

Вышеупомянутый калькулятор пролета стальной балки — это универсальный инструмент для проектирования конструкций, используемый для расчета изгибающего момента в алюминиевой, деревянной или стальной балке.Его также можно использовать в качестве калькулятора несущей способности балки, используя его в качестве калькулятора напряжения изгиба или напряжения сдвига. Он способен выдерживать до 2 различных сосредоточенных точечных нагрузок, 2 распределенных нагрузки и 2 момента. Распределенные нагрузки могут быть расположены так, чтобы они были равномерно распределенными нагрузками (UDL), треугольными распределенными нагрузками или трапециевидными распределенными нагрузками. Все нагрузки и моменты могут иметь направление как вверх, так и вниз по величине, что должно учитывать наиболее распространенные ситуации анализа балок.Расчет изгибающего момента и поперечной силы может занять до 10 секунд, и обратите внимание, что вы будете перенаправлены на новую страницу с реакциями, диаграммой поперечной силы и диаграммой изгибающего момента балки.

Одна из самых мощных функций — использование его в качестве калькулятора отклонения балки (или калькулятора смещения балки). Это можно использовать для наблюдения расчетного прогиба балки с опорой или консольной балки. Возможность добавлять формы сечения и материалы делает его полезным в качестве калькулятора деревянных балок или в качестве калькулятора стальных балок для проектирования балок lvl или i.На данный момент эта функция доступна в SkyCiv Beam, который имеет гораздо больше функций для проектирования деревянных, бетонных и стальных балок.

SkyCiv предлагает инженерам широкий спектр программного обеспечения для структурного анализа и проектирования облачных вычислений. Как постоянно развивающаяся технологическая компания, мы стремимся внедрять инновации и улучшать существующие рабочие процессы, чтобы сэкономить время инженеров в их рабочих процессах и проектах.

Размеры стальной балки

| SkyCiv

Наличие хорошего ресурса для размеров двутавровых балок очень важно при проектировании конструкций.Для нас, как инженера-строителя, важно определить секции, которые не только безопасны, но и рентабельны. С помощью приведенной ниже таблицы размеров стальных балок SkyCiv стремится получить бесплатный ресурс, к которому можно будет получить доступ в любое время через браузер. Приведенная ниже таблица размеров стальных балок поможет инженерам-строителям найти подходящий размер и форму, которые вам нужны для вашего проекта. Эти свойства важны при проверке прочности секции, что является само определение конструкции конструкции.

Таблица размеров стальных балок — это интерактивная таблица, в которой перечислены размерные и геометрические свойства сечения. Эти свойства могут помочь инженерам найти желаемый стальной профиль, который они ищут. Просто выберите систему единиц измерения, библиотеку, прежде чем выбирать форму, чтобы отобразить размеры балки этой формы. Библиотека содержит разделы из Австралии, США, Канады, Великобритании и Европы. Программное обеспечение извлекает размеры балки непосредственно из базы данных Structural 3D, которая является основным программным обеспечением для трехмерного структурного анализа платформы SkyCiv, которое также можно использовать в веб-браузере.Эта информация обычно требует, чтобы пользователь держал под рукой ручные или стальные диаграммы балок, что может быть неудобно, т. Е. Если у пользователя есть копия. Хотя приобретение руководства по стали требует денежных затрат, мы должны еще раз подчеркнуть, что справочная таблица SkyCiv легко доступна здесь бесплатно.

Некоторые из размеров, которые может отображать этот инструмент:

  • Размеры двутавровой балки
  • Размеры S-образной балки
  • Квадрат полый / HSS Размеры
  • Круглые размеры из быстрорежущей стали
  • Размеры балки с широким фланцем
  • Т-образная балка Размеры
  • Размеры каналов
  • Размеры уголков
  • Имперские и метрические размеры балки

Приведенные выше размеры стальных секций должны дать пользователю возможность легко получить доступ к свойствам элементов и размерам часто используемых секций в различных библиотеках по всему миру.Мы надеемся, что инженеры найдут ссылки на эти размеры и размеры стальных балок, которые будут полезны для их рабочего процесса. Опять же, в настоящее время существуют размеры и размеры стали для профилей из Австралии, США, Великобритании, Европы и Канады. Если конкретная библиотека, которую вы используете, недоступна, вы можете связаться с нами здесь. Мы открыты для улучшения и расширения нашей базы данных. Еще одним замечательным аспектом этого инструмента является то, что он может преобразовывать размеры балки из метрических в британские и наоборот. Это экономит время инженера при работе с единичными системами и снижает риск ошибки в расчетах.

Свойства сечения, отображаемые в приведенной выше таблице, включая площадь поперечного сечения (A), полярный момент инерции (J), момент площади (Iz, Iy), модуль упругости сечения и постоянную деформации (Iw). Эти результаты чрезвычайно важны при выборе конструкционной стали для конструкций балок и колонн. Это свойства, которые контролируют количество и тип силы, которую может принять стальной элемент.

Здесь, в SkyCiv, у нас есть ряд программного обеспечения (бесплатного и платного), которое позволяет инженерам моделировать и проектировать свои конструкции.Наш калькулятор свободных балок — это простой в использовании калькулятор, который помогает анализировать консольные балки и балки с простой опорой. Вы также можете воспользоваться нашим калькулятором ферм для быстрого анализа 2D ферм. Для небольших 2D-рам вы можете воспользоваться нашим бесплатным калькулятором несущих рам. Для типичных форм из стали нестандартных размеров калькулятор свободного момента инерции является хорошим средством для определения их геометрических характеристик и характеристик сечения.

SkyCiv предлагает инженерам широкий спектр программного обеспечения для структурного анализа и проектирования облачных вычислений.Как постоянно развивающаяся технологическая компания, мы стремимся внедрять инновации и улучшать существующие рабочие процессы, чтобы сэкономить время инженеров в их рабочих процессах и проектах.

Бесплатный калькулятор луча | ClearCalcs

Как использовать калькулятор свободного луча

Калькулятор луча ClearCalcs позволяет пользователю ввести геометрию и нагрузку луча для анализа за несколько простых шагов. Затем он определяет изгибающий момент, диаграммы сдвига и прогиба, а также максимальные требования, используя мощный механизм анализа методом конечных элементов.

Регистрация учетной записи ClearCalcs откроет дополнительные расширенные функции для проектирования и анализа балок и множества других структурных элементов. ClearCalcs позволяет проектировать из стали, бетона и дерева в соответствии со стандартами Австралии, США и ЕС.

Лист разделен на три основных раздела:

  1. «Ключевые свойства», где пользователь вводит геометрию выбранного сечения и опор балки.
  2. «Нагрузки», где можно вводить распределенные, точечные и приложенные моментные нагрузки,
  3. «Сводка», в котором отображаются основные выходные данные и диаграммы.

Раздел «Комментарии» также включен для того, чтобы пользователь мог оставить какие-либо конкретные примечания по дизайну. Щелчок по любой из меток ввода / свойства дает описательное справочное объяснение.

1. Свойства входных клавиш

Свойства балки и сечения задаются путем ввода непосредственно в поля ввода.

Длина балки — это общая длина балки, включая все пролеты балки, в мм или футах.

Модуль Юнга установлен на значение по умолчанию 200000 МПа или 29000 тысяч фунтов на квадратный дюйм для конструкционной стали, но может быть изменен Пользователь.

Площадь поперечного сечения зависит от выбранного сечения балки и по умолчанию соответствует значениям для обычной стальной балки.

Второй момент площади (или момент инерции) также зависит от выбранного сечения балки и снова по умолчанию соответствует свойствам обычной стальной балки.

Свойства E, A и Ix для других секций балки можно получить из библиотеки свойств секций ClearCalcs. Кроме того, вы можете создать свой собственный раздел, используя наш бесплатный калькулятор момента инерции.

Положение опор слева позволяет пользователю вводить любое количество опор и указывать их положение по длине балки. Тип опоры может быть закрепленным (фиксированный в перемещении, свободном вращении) или фиксированным (фиксированный как при перемещении, так и при повороте) и выбирается из раскрывающегося меню. Требуется минимум одна фиксированная опора или две штифтовые опоры.

Вычислитель балки также учитывает пролет консолей на каждом конце, поскольку положение первой опоры не обязательно должно быть равно 0 мм, а положение последней опоры не обязательно должно быть равно длине балки.

Реакции на каждой из опор автоматически обновляются по мере добавления, изменения или удаления опор в зависимости от указанной нагрузки.

2. Входные нагрузки

Калькулятор поддерживает различные типы нагрузок, которые можно применять в комбинации. Каждой загрузке может быть присвоено имя пользователем.

Знаковое обозначение, используемое для нагружения (показаны положительные значения):

Распределенные нагрузки указываются в единицах силы на единицу длины, кН / м или plf, вдоль балки и могут применяться между любыми двумя точками.В калькуляторе можно использовать два разных типа:

Равномерная нагрузка имеет постоянную величину по всей длине приложения. Следовательно, начальная и конечная величины, указанные пользователем, должны быть одинаковыми.

Линейные нагрузки имеют переменную величину по длине приложения. Различные начальные и конечные величины должны быть указаны пользователем, и они могут использоваться для представления треугольных или трапециевидных нагрузок.

Точечные нагрузки указываются в единицах силы, кН или тысячах фунтов, и площади, приложенной в дискретных точках вдоль балки.Например, они могут представлять реакции других элементов, соединенных с балкой. Пользователь вводит имя, величину и местоположение слева от луча.

В приведенном ниже примере диаграммы из сводного раздела показана двухпролетная неразрезная балка с линейно распределенной нагрузкой на заплату и точечной нагрузкой.

3. Итоговые результаты вычислений

После задания нагрузки и геометрии калькулятор автоматически использует механизм конечно-элементного анализа ClearCalcs для определения моментов, поперечных сил и прогибов.Максимальные значения каждого из них выводятся как «Требование момента» , «Требование сдвига» и «Прогиб» вместе с диаграммами по длине балки.

Положительные значения означают отклонение вниз, а отрицательные значения — отклонение вверх. Знаковое соглашение, используемое на диаграммах поперечной силы и изгибающего момента, следующее (показаны положительные значения):

Использование курсора для наведения курсора на любую точку на диаграммах изгибающего момента, поперечной силы или прогиба дает конкретные значения в этом месте вдоль балки.В приведенном ниже примере показаны выходные параметры для двухпролетной неразрезной балки с линейно распределенной коммутационной нагрузкой и точечной нагрузкой.

Напряжение и прогиб балки | MechaniCalc

ПРИМЕЧАНИЕ. Эта страница использует JavaScript для форматирования уравнений для правильного отображения. Пожалуйста, включите JavaScript.


Многие конструкции можно представить как прямую балку или как набор прямых балок. По этой причине анализ напряжений и прогибов в балке является важной и полезной темой.

В этом разделе рассматриваются поперечная сила и изгибающий момент в балках, диаграммы сдвига и момента, напряжения в балках и таблица общих формул прогиба балок.

Содержание

Сила сдвига и изгибающий момент

Чтобы найти поперечную силу и изгибающий момент по длине балки, сначала решите внешние реакции при граничных условиях. Например, консольная балка ниже имеет приложенную силу, показанную красным, а реакции показаны синим цветом при фиксированном граничном условии:

После того, как были решены внешние реакции, сделайте разрезы секций по длине балки и решите реакции на каждом разрезе секции.Пример разреза показан на рисунке ниже:

Когда балка разрезается по сечению, при вычислении реакций можно учитывать любую сторону балки. Выбранная сторона не влияет на результаты, поэтому выберите наиболее легкую. На рисунке выше выбрана сторона балки справа от разреза. Реакции на разрезе показаны синими стрелками.

Подписать Конвенцию

Знаки сдвига и момента важны.Знак определяется после того, как сделан разрез и решены реакции для части балки на одной стороне разреза. Сила сдвига в разрезе секции считается положительной, если она вызывает вращение выбранной секции балки по часовой стрелке, и отрицательной, если вызывает вращение против часовой стрелки. Изгибающий момент в разрезе секции считается положительным, если он сжимает верхнюю часть балки и удлиняет нижнюю часть балки (т.е. если он заставляет балку «улыбаться»).

Исходя из этого соглашения о знаках, поперечная сила в разрезе секции на рисунке выше является положительной, поскольку она вызывает вращение выбранной секции по часовой стрелке.Момент отрицательный, так как он сжимает нижнюю часть балки и удлиняет верхнюю часть (т.е. заставляет балку «хмуриться»).


Ознакомьтесь с нашим калькулятором балок, основанным на методике, описанной здесь.

  • Расчет напряжений и прогибов в прямых балках
  • Строит диаграммы сдвига и момента
  • Может указывать любую конфигурацию ограничений, сосредоточенных сил и распределенных сил

Диаграммы сдвига и момента

Сдвиговый и изгибающий моменты в балке обычно выражаются диаграммами.Диаграмма сдвига показывает сдвиг по длине балки, а диаграмма моментов показывает изгибающий момент по длине балки. Эти диаграммы обычно показаны сложенными друг на друга, и комбинация этих двух диаграмм представляет собой диаграмму момента сдвига. Диаграммы момента сдвига для некоторых общих конечных условий и конфигураций нагрузки показаны в таблицах прогиба балок в конце этой страницы. Пример диаграммы момента сдвига показан на следующем рисунке:

Общие правила построения диаграмм момента сдвига приведены в таблице ниже:

Диаграмма сдвига Схема моментов
  • Точечные нагрузки вызывают вертикальный скачок на диаграмме сдвига.Направление прыжка совпадает со знаком точечной нагрузки.
  • Равномерно распределенные нагрузки дают прямую наклонную линию на диаграмме сдвига. Наклон линии равен величине распределенной нагрузки.
  • Диаграмма сдвига горизонтальна для расстояний вдоль балки без приложенной нагрузки.
  • Сдвиг в любой точке балки равен наклону момента в этой же точке:
  • Диаграмма моментов представляет собой прямую наклонную линию для расстояний вдоль балки без приложенной нагрузки.Наклон линии равен величине сдвига.
  • Равномерно распределенные нагрузки приводят к параболической кривой на диаграмме моментов.
  • Максимальные / минимальные значения момента возникают там, где линия сдвига пересекает ноль.
  • Момент в любой точке балки равен площади под диаграммой сдвига до этой точки:

    M = ∫ V dx

Напряжения изгиба в балках

Изгибающий момент M по длине балки можно определить по диаграмме моментов.Изгибающий момент в любом месте балки затем можно использовать для расчета изгибающего напряжения по поперечному сечению балки в этом месте. Изгибающий момент изменяется по высоте поперечного сечения в соответствии с приведенной ниже формулой изгиба :

где M — изгибающий момент в интересующем месте по длине балки, I c — центроидный момент инерции поперечного сечения балки, а y — расстояние от нейтральной оси балки до интересующей точки по высоте. поперечного сечения.Отрицательный знак указывает, что положительный момент приведет к сжимающему напряжению выше нейтральной оси.

Напряжение изгиба равно нулю на нейтральной оси балки, которая совпадает с центром тяжести поперечного сечения балки. Напряжение изгиба линейно увеличивается от нейтральной оси до максимальных значений на крайних волокнах вверху и внизу балки.

Максимальное напряжение изгиба определяется как:

где c — центроидное расстояние поперечного сечения (расстояние от центроида до крайнего волокна).

Если балка асимметрична относительно нейтральной оси, так что расстояния от нейтральной оси до верха и низа балки не равны, максимальное напряжение будет возникать в самом дальнем от нейтральной оси месте. На рисунке ниже растягивающее напряжение в верхней части балки больше, чем сжимающее напряжение в нижней части.

Модуль упругости поперечного сечения объединяет центроидный момент инерции I c и центральное расстояние c:

Преимущество модуля сечения состоит в том, что он характеризует сопротивление сечения изгибу одним членом.Модуль сечения можно подставить в формулу изгиба для расчета максимального напряжения изгиба в поперечном сечении:


Ознакомьтесь с нашим калькулятором балок, основанным на методике, описанной здесь.

  • Расчет напряжений и прогибов в прямых балках
  • Строит диаграммы сдвига и момента
  • Может указывать любую конфигурацию ограничений, сосредоточенных сил и распределенных сил

Напряжения сдвига в балках

Сила сдвига V по длине балки может быть определена из диаграммы сдвига.Сила сдвига в любом месте вдоль балки затем может использоваться для расчета напряжения сдвига по поперечному сечению балки в этом месте. Среднее напряжение сдвига по поперечному сечению определяется как:

Напряжение сдвига меняется по высоте поперечного сечения, как показано на рисунке ниже:

Напряжение сдвига равно нулю на свободных поверхностях (вверху и внизу балки) и максимально в центре тяжести. Уравнение для касательного напряжения в любой точке, расположенной на расстоянии y 1 от центра тяжести поперечного сечения, определяется следующим образом:

где V — поперечная сила, действующая в месте поперечного сечения, I c — центроидный момент инерции поперечного сечения, а b — ширина поперечного сечения.Все эти термины являются константами. Член Q — это первый момент области, ограниченной интересующей точкой и крайним волокном поперечного сечения:

Напряжения сдвига для нескольких общих поперечных сечений обсуждаются в следующих разделах.

Напряжения сдвига в прямоугольном сечении

Распределение касательного напряжения по высоте прямоугольного поперечного сечения показано на рисунке ниже:

Первый момент площади в любой заданной точке y 1 по высоте поперечного сечения вычисляется по формуле:

Максимальное значение Q приходится на нейтральную ось балки (где y 1 = 0):

Напряжение сдвига в любой заданной точке y 1 по высоте поперечного сечения рассчитывается по формуле:

где I c = b · h 3 /12 — центроидный момент инерции поперечного сечения.Максимальное напряжение сдвига возникает на нейтральной оси балки и рассчитывается по формуле:

где A = b · h — площадь поперечного сечения.

Обратите внимание, что максимальное напряжение сдвига в поперечном сечении на 50% выше, чем среднее напряжение V / A.

Напряжения сдвига в круглых сечениях

Круглое поперечное сечение показано на рисунке ниже:

Уравнения для касательного напряжения в балке были получены с использованием предположения, что касательное напряжение по ширине балки является постоянным.Это предположение справедливо в центре тяжести кругового поперечного сечения, хотя нигде больше не действует. Следовательно, хотя распределение напряжения сдвига по высоте поперечного сечения не может быть легко определено, максимальное напряжение сдвига в сечении (возникающее в центре тяжести) все же может быть вычислено. Максимальное значение первого момента Q, возникающего в центроиде, определяется как:

Затем максимальное напряжение сдвига рассчитывается по формуле:

где b = 2r — диаметр (ширина) поперечного сечения, I c = πr 4 /4 — центроидный момент инерции, а A = πr 2 — площадь поперечного сечения.

Напряжения сдвига в круглых сечениях трубы

Круглое поперечное сечение трубы показано на рисунке ниже:

Максимальное значение первого момента Q, возникающего в центроиде, определяется как:

Затем максимальное напряжение сдвига рассчитывается по формуле:

где b = 2 (r o — r i ) — эффективная ширина поперечного сечения, I c = π (r o 4 — r i 4 ) / 4 — центроидный момент инерции, а A = π (r o 2 — r i 2 ) — площадь поперечного сечения.

Напряжения сдвига в двутаврах

Распределение напряжения сдвига вдоль стенки двутавровой балки показано на рисунке ниже:

Уравнения для касательного напряжения в балке были получены с использованием предположения, что касательное напряжение по ширине балки является постоянным. Это предположение справедливо для стенки двутавровой балки, но неверно для полок (особенно там, где стенка пересекает полки). Однако стенка двутавровой балки принимает на себя подавляющую часть силы сдвига (примерно 90% — 98%, согласно Гиру), и поэтому можно консервативно предположить, что стенка несет всю силу сдвига.

Первый момент площади перемычки двутавровой балки определяется по формуле:

Напряжение сдвига вдоль стенки двутавровой балки определяется по формуле:

где t w — толщина стенки, а I c — центроидный момент инерции двутавровой балки:

Максимальное значение напряжения сдвига возникает на нейтральной оси (y 1 & равно; 0), а минимальное значение напряжения сдвига в полотне возникает на внешних волокнах полотна, где оно пересекает фланцы y 1 & equals; & pm; h w /2):


Ознакомьтесь с нашим калькулятором балок, основанным на методике, описанной здесь.

  • Расчет напряжений и прогибов в прямых балках
  • Строит диаграммы сдвига и момента
  • Может указывать любую конфигурацию ограничений, сосредоточенных сил и распределенных сил

Таблицы прогиба балок

В таблицах ниже приведены уравнения прогиба, наклона, сдвига и момента вдоль прямых балок для различных конечных условий и нагрузок. Вы можете найти исчерпывающие таблицы в таких источниках, как Гир, Линдебург и Шигли.Однако приведенные ниже таблицы охватывают большинство распространенных случаев.

Консольные балки

Балки с простой опорой

Несъемные несущие балки


Подпишитесь, чтобы получать обновления о последних улучшениях:


Список литературы

  1. Будинас-Нисбетт, «Машиностроительный проект Шигли», 8-е изд.
  2. Гир, Джеймс М., «Механика материалов», 6-е изд.
  3. Линдебург, Майкл Р., «Справочное руководство по машиностроению для экзамена на физическую форму», 13-е изд.
  4. «Руководство по анализу напряжений», Лаборатория динамики полета ВВС, октябрь 1986 г.

Калькулятор луча | MechaniCalc

Калькулятор балки позволяет анализировать напряжения и прогибы в прямых балках.

Опции

Пример загрузки

Очистить все данные


Входы

Введите данные балки, затем нажмите кнопку «Рассчитать результаты»:

Добавить ограничение

Удалить ограничение


Невозможно отобразить сюжет — браузер устарел.

Рассчитать результаты

Предупреждение — Перед решением необходимо исправить следующее:


Дисплейные блоки


Результаты

Результаты анализа пучка подробно описаны ниже. Задача решалась в виде конечно-элементной модели с использованием балочных элементов. Для получения дополнительной информации о том, как были получены эти результаты, обратитесь к справочнику по конечно-элементному анализу и справочнику по напряжению и прогибу балки.

Обзор результатов

Максимальный прогиб и наклон приведены ниже:

Значение Место нахождения
Максимальное отклонение:
Максимальный наклон:

Схема свободного тела (FBD) и деформированная сетка показаны ниже.


Невозможно отобразить сюжет — браузер устарел.


Невозможно отобразить сюжет — браузер устарел.



См. Полную информацию о результатах на других вкладках (выше).

Обзор модели

Модель с приложенными силами и ограничениями показана ниже:


Невозможно отобразить сюжет — браузер устарел.


Свойства материала

Материал:

Имущество Значение
Предел текучести
Максимальная сила
Модуль упругости
Коэффициент Пуассона

Свойства поперечного сечения

Поперечное сечение:

Имущество Значение
Высота (Y)
Ширина (X)
Толщина стенки
Толщина фланца
Площадь
Центроидное расстояние
(в направлении первичного изгиба)
Момент инерции, центроидный угол
(относительно оси первичного изгиба)

Диаграмма момента сдвига

Диаграммы сдвига и момента показаны ниже.Соблюдаются стандартные условные обозначения для диаграмм момента сдвига:

  • Сдвиг: положительный сдвиг вызывает вращение балки по часовой стрелке, отрицательный сдвиг вызывает вращение против часовой стрелки.
  • Момент: Положительный момент сжимает верхнюю часть балки и удлиняет нижнюю часть балки (т.е. заставляет балку «улыбаться»).

Невозможно отобразить сюжет — браузер устарел.

Невозможно отобразить сюжет — браузер устарел.


Невозможно отобразить сюжет — браузер устарел.


Графики напряжений

Графики напряжений показаны ниже.


Невозможно отобразить сюжет — браузер устарел.

Невозможно отобразить сюжет — браузер устарел.


Невозможно отобразить сюжет — браузер устарел.

Невозможно отобразить сюжет — браузер устарел.

Напряжения рассчитываются на основе следующих уравнений:

Осевое напряжение Напряжение сдвига Напряжение изгиба Напряжение фон Мизеса

Графики прогиба

Графики прогиба показаны ниже.Условные обозначения прогибов:

.
  • X: положительный направо, отрицательный налево
  • Y: положительный вверх, отрицательный вниз
  • Наклон: линейка правой руки (положительное значение против часовой стрелки, отрицательное по часовой стрелке)

Невозможно отобразить сюжет — браузер устарел.

Невозможно отобразить сюжет — браузер устарел.


Невозможно отобразить сюжет — браузер устарел.

Невозможно отобразить сюжет — браузер устарел.


Эта проблема была решена в виде конечно-элементной модели. На этой вкладке представлены результаты для отдельных узлов и элементов модели.

На приведенном ниже графике показана сетка с номерами элементов , помеченными:

Невозможно отобразить сюжет — браузер устарел.



Узловые результаты

Ниже приведены результаты для каждого узла. Следует отметить несколько моментов:

  • Определенные узлы связаны с точками, и для этих узлов указывается номер связанной точки.
  • Сначала перечислены все узлы, связанные с точками, за ними следуют узлы, созданные как часть процесса построения сетки.
  • Для ограниченных степеней свободы могут существовать внешние реакции. Любые узлы, не имеющие ограничений, не будут иметь внешних реакций.


Элементарные результаты

Ниже приведены результаты для каждого элемента. Следует отметить несколько моментов:

  • Каждый элемент состоит из 2 узлов. В таблице эти узлы обозначаются как «Узел 1» и «Узел 2».
  • Внутренние реакции даны в терминах глобальной системы координат (т.е. X и Y), а также в локальной системе координат (т.е. «осевой» вдоль оси элемента, «сдвиг» перпендикулярно элементу).



Загрузить результаты в Excel

Загрузите файл Excel на свой компьютер, содержащий узловые и элементарные результаты.


Скачать отчет

Сохраните отформатированный документ Word на свой компьютер с подробным описанием входных данных и результатов анализа.


Скачать входной файл

Сохранить все входные данные в файл. Позже вы можете загрузить этот файл, чтобы продолжить с того места, где вы остановились.



Нужна дополнительная функциональность?

Зарегистрируйте учетную запись, чтобы получить полный доступ ко всем калькуляторам и другому контенту. Типы подписки описаны ниже вместе с преимуществами каждого из них.

  • Цена
  • Доступ к калькуляторам
  • Логин
  • Создание материалов
  • Создание сечений
  • Сохранение файлов
  • Отчетность
  • Бесплатно
  • Ограничено

    Limited Доступ к калькуляторам

  • Предварительно определенные Поперечные сечения

  • Учить больше »
  • 39 долларов США.99 / месяц
    249,99 долларов США в год
  • Полный

    Полный Доступ к калькуляторам

  • Плавающие лицензии

    Плавающие лицензии

  • Учить больше »
  • Зарегистрироваться сейчас

Калькулятор изгибающего момента и поперечного усилия

Диаграмма момента изгиба.com — это бесплатный онлайн-калькулятор, который создает диаграммы изгибающего момента (BMD) и диаграммы поперечного усилия (SFD) для большинства простых балок. Калькулятор полностью настраивается для работы с большинством балок; эта функция недоступна в большинстве других калькуляторов. Программное обеспечение работает на базе SkyCiv, предлагая мощное программное обеспечение для структурного анализа и проектирования в облаке.

Инструмент полностью функциональный, поэтому посетите наше Бесплатное программное обеспечение Beam, чтобы начать работу! Он будет работать со всеми опорными, определяющими балками и способен воспринимать точечные нагрузки, сосредоточенные моменты и распределенные нагрузки.Кроме того, его можно легко настраивать и настраивать, чтобы вы могли создавать свои собственные лучи. Это чрезвычайно точный инструмент и, в отличие от современных калькуляторов, очень удобный. Это чрезвычайно полезный инструмент для студентов университетов, колледжей и старшеклассников, которым утомительно приходится перерисовывать BMD и SFD для заданий и практических / учебных вопросов.

У нас также есть учебная страница, которая поможет студентам университетов с расчетами, ожидаемыми в их инженерной степени, а также студентам старших классов.Эти студенты могут научиться рассчитывать и создавать диаграммы поперечной силы и изгибающего момента, и мы понимаем, что процесс анализа балки иногда может быть трудным, поэтому мы предоставили простое пошаговое руководство по расчету диаграмм изгибающего момента и поперечной силы. Включены простые уравнения и формулы изгибающего момента, которые хорошо помогают в ваших расчетах. Существуют также примеры и генераторы случайных балок, которые позволят вам поэкспериментировать с тем, как различные нагрузки влияют на расчет балки, а также на поперечную силу и изгибающий момент балки.

Схема

Bending Moment Diagram разработана командой SkyCiv Engineering, которая предлагает студенческие и профессиональные пакеты, которые предоставляют пользователям доступ к разнообразному программному обеспечению для проектирования конструкций для выполнения работы. Все учетные записи основаны на подписке, поэтому вы можете ежемесячно оплачивать программное обеспечение по мере необходимости! Больше никаких проблем с установкой, загрузкой или лицензированием!

Калькулятор отклонения балки

Этот калькулятор отклонения балки поможет вам определить максимальное отклонение балки для балок с простой опорой и консольных балок, несущих простых конфигураций нагрузки .Вы можете выбрать один из нескольких типов нагрузки, которые могут воздействовать на балку любой длины по вашему желанию. Величина и расположение этих нагрузок влияют на то, насколько балка изгибается. В этом калькуляторе отклонения балки вы узнаете о различных формулах отклонения балки , используемых для расчета прогибов балок без опоры и изгибов консольных балок. Вы также узнаете, как модуль упругости балки и момент инерции ее поперечного сечения влияют на расчетный максимальный прогиб балки.

Что такое прогиб балки и изгиб балки

В строительстве мы обычно используем каркасных конструкций , которые удерживаются на месте фундаментом в земле. Эти каркасные конструкции подобны каркасам зданий, домов и даже мостов. В раме мы называем вертикальное обрамление колонны , а горизонтальные балки . Балки — это длинные элементы конструкции, которые несут нагрузки, создаваемые горизонтальными плитами конструкций, включая перекрытия и крыши.

Когда балки несут слишком тяжелые для них нагрузки, они начинают гнуться. Мы называем величину изгиба балки , прогиб балки . Отклонение балки — это вертикальное смещение точки вдоль центра тяжести балки. Мы также можем рассматривать поверхность балки как опорную точку, если нет изменений в высоте или глубине балки во время изгиба.

Как рассчитать максимальный прогиб балки

Мы снабдили наш калькулятор прогиба балки формулами, которые инженеры и студенты-инженеры используют для быстрого определения максимального прогиба, который будет испытывать конкретная балка из-за нагрузки, которую она несет.Однако эти формулы могут решать только простые нагрузки и их комбинацию. Мы составили для вас таблицы этих формул, как показано ниже:

Формулы прогиба балок с простой опорой

Формулы прогиба консольной балки


Метод наложения

Для расчета максимального прогиба балки с комбинацией нагрузок мы можем использовать метод наложения . Метод наложения утверждает, что мы можем приблизительно оценить полное отклонение балки, сложив вместе все отклонения, вызванные каждой конфигурацией нагрузки.Однако этот метод дает нам лишь приблизительное значение фактического максимального прогиба. Расчет сложных нагрузок потребует от нас использования так называемого метода двойного интегрирования .

Жесткость балки

Для расчета прогиба балки необходимо знать жесткость балки и величину силы или нагрузки, которые могут повлиять на изгиб балки. Мы можем определить жесткость балки, умножив модуль упругости балки , E , на ее момент инерции , I .Модуль упругости зависит от материала балки. Чем выше модуль упругости материала, тем больше прогиб может выдержать огромные нагрузки, прежде чем достигнет предела разрушения. Модуль упругости бетона составляет 15-50 ГПа (гигапаскалей), а у стали — около 200 ГПа и выше. Эта разница в значениях модуля упругости показывает, что бетон может выдерживать лишь небольшой прогиб и трескается быстрее, чем сталь.

Вы можете узнать больше о модуле упругости, воспользовавшись нашим калькулятором напряжений.С другой стороны, чтобы определить момент инерции для определенного поперечного сечения балки, вы можете воспользоваться нашим калькулятором момента инерции. Момент инерции представляет собой величину сопротивления материала вращательному движению. Момент инерции зависит от размеров поперечного сечения материала.

Момент инерции также зависит от оси вращения материала. Чтобы лучше понять эту концепцию, давайте рассмотрим поперечное сечение прямоугольной балки шириной 20 см и высотой 30 см.Используя формулы, которые вы также можете увидеть в нашем калькуляторе момента инерции, мы можем вычислить значения момента инерции этого поперечного сечения следующим образом:

Iₓ = ширина * высота³ / 12
= 20 * (30³) / 12
= 45000 см⁴

Iᵧ = высота * ширина³ / 12
= 30 * (20³) / 12
= 20,000 см⁴

Обратите внимание на два значения момента инерции. Это потому, что мы можем рассматривать изгиб балки по вертикали (по оси x, то есть Iₓ) или по горизонтали (по оси y, то есть Iᵧ).Поскольку мы учитываем отклонение балки при ее вертикальном изгибе, для расчетов всегда нужно использовать Iₓ . Полученные нами значения говорят нам о том, что балку сложнее согнуть при вертикальной нагрузке и легче согнуть при горизонтальной нагрузке. Эта разница в значениях момента инерции является причиной того, что мы видим балки в этой конфигурации, в которой ее высота больше, чем ширина.

Понимание формул прогиба балки

Теперь, когда мы знаем концепции модуля упругости и момента инерции, теперь мы можем понять, почему эти переменные являются знаменателями в наших формулах отклонения балки.Формулы показывают, что чем жестче балка, тем меньше будет ее прогиб. Однако, изучив наши формулы, мы также можем сказать, что длина балки также напрямую влияет на прогиб балки. Чем длиннее балка, тем больше она может изгибаться и тем больше может быть прогиб.

Нагрузки, с другой стороны, влияют на отклонение балки двумя способами: направление отклонения и величина отклонения . Нисходящие нагрузки склонны отклонять балку вниз.Нагрузки могут быть в виде точечной нагрузки, линейного давления или моментной нагрузки. Формулы в этом калькуляторе ориентированы только на нисходящие или восходящие направления для точечной нагрузки и распределенных нагрузок. Распределенные нагрузки аналогичны давлению, но учитывают только длину балки, а не ширину балки. Формулы в этом калькуляторе также учитывают момент или крутящий момент нагрузки как по часовой стрелке, так и против часовой стрелки. Просто проконсультируйтесь по направлениям стрелок на соответствующем изображении формулы, чтобы выяснить, в каком направлении имеется положительное значение нагрузки.

Пример расчета прогиба балки

Для примера расчета прогиба балки рассмотрим простую деревянную скамью с ножками на расстоянии 1,5 метра друг от друга в их центрах. Допустим, у нас есть доска из восточной белой сосны толщиной 4 см и шириной 30 см, которая служит сиденьем для этой скамейки. Мы можем рассматривать это сиденье как балку, которая отклоняется, когда кто-то садится на скамейку. Зная размеры этого сиденья, мы можем вычислить его момент инерции, как в нашем примере выше.Поскольку нам нужно рассчитать Iₓ, его момент инерции будет:

Iₓ = ширина * высота³ / 12
= 30 * (4³) / 12
= 160,0 см⁴ или 1,6x10⁻⁶ м⁴

Сосна белая восточная имеет модуль упругости 6,800 МПа (6,8x10⁹ Па) , что является значением, которое мы получили из Справочника по древесине. Вы также можете легко получить значение модуля упругости для других материалов, таких как сталь и бетон, в Интернете или в местной библиотеке.Теперь, когда мы знаем эти значения, давайте рассмотрим нагрузку, которую будет нести этот стенд. Предположим, что ребенок 400 N сидит в центре скамейки. Теперь мы можем рассчитать прогиб сиденья скамейки из-за точечной нагрузки в его центре:

δₘₐₓ = P * L³ / (48 * E * I)
δₘₐₓ = (400 Н) * (1,5 м) ³ / (48 * 6,8x10⁹ Па * 1,6x10⁻⁶ м⁴)
δₘₐₓ = 0,002585 m = 2,5850 мм

Это означает, что многоместное сиденье прогнется примерно на 2.6 миллиметров на от исходного положения, когда ребенок сидит посередине скамейки.

Если вы нашли эту тему интересной и хотели бы узнать больше о прочности материалов, вам также может понравиться наш калькулятор запаса прочности. Вы также можете воспользоваться нашим конвертером силы, если хотите изучить различные единицы измерения точечных нагрузок и расчета сил.

alexxlab

Related Posts

Добавить комментарий

Ваш адрес email не будет опубликован. Обязательные поля помечены *